Health Assessment Lab Manual Questions

¡Supera tus tareas y exámenes ahora con Quizwiz!

While interviewing an adult client about her nutrition habits the nurse should A. ask the client for a 3-day recall of food intake. B. review the food pyramid with the client. C. ask the client about limitations to activity. D. encourage the client to drink three to four glasses of water daily.

,B. review the food pyramid with the client.

The nurse has assessed the breath sounds of an adult client. The best way for the nurse to document these findings on a client is to write A. "Bilateral lung sounds clear." B. "The client's lung sounds were clear on both sides." C. "Client's lung sounds were auscultated with stethoscope and were clear on both sides." D. "After listening to client's lung sounds both lungs appeared clear."

A. "Bilateral lung sounds clear."

A client visits the clinic for a routine examination. The client tells the nurse she has become constipated because she is taking iron tablets prescribed for anemia. The nurse has instructed the client about the use of iron preparations and possible constipation. The nurse determines that the client has understood the instructions when she says A. "I can decrease the constipation if I eat foods high in fiber and drink water." B. "I should cut down on the number of iron tablets I am taking each day." C. "Constipation should decrease if I take the iron tablets with milk." D. "I should discontinue the iron tablets and eat foods that are high in iron."

A. "I can decrease the constipation if I eat foods high in fiber and drink water."

Cultural factors play an important role in the development of prostate cancer in men. Which culture has the highest prostate cancer rate? A. African American B. White American C. Italian D. Japanese

A. African American

The nurse is assessing a client who has been taking antibiotics for an infection for 10 days. The nurse observes whitish curd-like patches in the client's mouth. The nurse should explain to the client that these spots are most likely A. Candida albicans infection. B. Koplik's spots. C. leukoplakia. D. Fordyce spots.

A. Candida albicans infection.

The nurse is planning a presentation to a group of high school students about the risk factors for oral cancer. Which of the following should be included in the nurse's plan? A. Diets low in fruits and vegetables are a possible risk factor for oral cancer. B. About 40% of all cancers occur in the lips mouth, and tongue. C. The incidence of oral cancers is higher in women than in men. D. Most oral cancers are detected in people in their 70s.,

A. Diets low in fruits and vegetables are a possible risk factor for oral cancer.

The nurse is caring for an adult client who tells the nurse "For weeks now I just can't get to sleep at night because of all the noise in my neighborhood." An actual nursing diagnosis for this client is A. Fatigue related to excessive noise levels as manifested by client's statements of chronic fatigue. B. Sleep deprivation related to noisy neighborhood and inability to sleep. C. Chronic fatigue syndrome related to excessive levels of noise in neighborhood. D. Readiness for enhanced sleep related to control of noise level in the home.,

A. Fatigue related to excessive noise levels as manifested by client's statements of chronic fatigue.

What is the most common measurement used to determine abdominal visceral fat? A. Waist circumference B. Body mass index (BMI) C. Subcutaneous fat determination D. Triceps skin-fold thickness

A. Waist circumference

Th e nurse is assessing a dark-skinned client who has been transported to the emergency room by ambulance. When the nurse observes that the client's skin appears pale with blue-tinged lips and oral mucosa, the nurse should document the presence of A. a great degree of cyanosis. B. a mild degree of cyanosis. C. lupus erythematosus. D. hyperthyroidism.,

A. a great degree of cyanosis.

A risk factor for sinusitis in the frail elderly is A. a nasogastric feeding tube. B. an accumulation of ear wax. C. decreased ability to detect odors. D. conductive hearing loss.

A. a nasogastric feeding tube.

The nurse is beginning a health history interview with an adult client who expresses anger at the nurse. The best approach for dealing with an angry client is for the nurse to A. allow the client to ventilate his or her feelings. B. offer reasons why the client shouldn't feel angry. C. provide structure during the interview. D. refer the client to a different health care provider.

A. allow the client to ventilate his or her feelings.

The meibomian glands secrete A. an oily substailce to lubricate the eyes. B. sweat. C. hormones. D. clear liquid tears.

A. an oily substailce to lubricate the eyes.

menopausal phase of her life. The nurse should instruct the client that she may experience A. hot flashes. B. increased appetite. C. vaginal discharge. D. urinary frequency.

A. hot flashes.

The popliteal artery can be palpated at the A. knee. B. great toe. C. ankle. D. inguinal ligament.

A. knee.

The lymph nodes that are responsible for drainage from the arms are the A. lateral lymph nodes. B. central lymph nodes. C. anterior lymph nodes. D. posterior lymph nodes.

A. lateral lymph nodes

A client visits the clinic and tells the nurse that she had a mastectomy 2 years ago. The nurse should assess the client for A. lymphedema. B. Raynaud's disease. C. poor peripheral pulses. D. bruits over the radial artery.

A. lymphedema.

While assessing the musculoskeletal system of an adult client the nurse observes hard painless nodules over the distal interphalangeal joints. The nurse should document the presence of A. osteoarthritis. B. bursitis. C. tendonitis. D. rheumatoid arthritis.

A. osteoarthritis.

Sensory receptors in the ear that help to maintain both static and dynamic equilibrium are located in the semicircular canals and the A. vestibule. B. tympanic membrane. C. cone of light. D. eustachian tube.

A. vestibule.

The skin folds of the labia majora and the labia minora form a boat-shaped area termed the A. vestibule. B. corpus. C. Skcnc's glands. A D. urethral meatus.

A. vestibule.

Before beginning a physical assessment of a client he nurse should first A. wash both hands with soap and water. B. determine whether the client is anxious. C. ask the client to remove all clothing. D. request a family member to be present.,

A. wash both hands with soap and water.

The nurse has instructed a S5-yesr-old male client about the need for a stool test for occult blood. The nurse determines that the client understands the instructions when he says the test should be performed every A. year. B. 2 years. C. 3 years. D. 4 years.

A. year.

Which of the following is part of a Mental Status Exam? A. Eliciting reflexes. B. Evaluating level of consciousness. C. Evaluating pain sensations. D. Assessing cranial nerves.

B. Evaluating level of consciousness.

Based only on anthropometric measurements which set of clients listed below is at the greatest risk for diabetes and cardiovascular disease? A. Clients with a body mass index of 23 B. Females with 35 inches or greater waist circumference C. Males with 35 inches or greater waist circumference D. Clients with a body mass index of 20

B. Females with 35 inches or greater waist circumference

A neurologic clin~igea ssociated with normal aging is A. loss of long-tern~m emory. B. a decrease In reaction time. C. swaying or shuffling gait. D. a significant decline in judgment and cognition.

B. a decrease In reaction time

The functional reflex that allows the eyes to focus on near objects is termed A. pupillary reflex B. accommodation. C. refraction. D. indirect reflex.

B. accommodation.

While percussing an adult client during a physical examination the nurse can expect to hear flatness over the client's A. lungs. B. bone. C. liver. D. abdomen.

B. bone.

Two body systems that may be logically integrated and assessed at the same time are the A. eye and ear exams. B. eye exam and cranial nerves 11 111, IV, and VI. C. ear exam and cranial nerves IV, V1, and VIII. D. ear and nose exams.,

B. eye exam and cranial nerves 11, 111, IV, and VI.

A primary function of hair in the nose and eyelashes is to serve as a A. response to cold. B. filter for dust. C. pigment producer. D. response to fright.

B. filter for dust.

To palpate the spleen of an adult client the nurse should A. ask the client to exhale deeply. B. place the right hand below the left costal margin. C. point the fingers of the left hand downward. D. ask the client to remain in a supine position.

B. place the right hand below the left costal margin.

The nurse is planning to interview a client who has demonstrated manipulative behaviors during past clinic visits. During the interview process the nurse should plan to A. give the client rules with which he must agree to comply. B. provide structure and set limits with the client. C. tell the client that the nurse is aware of his past behaviors. D. approach the client in an authoritative manner.,

B. provide structure and set limits with the client.

During the interview of an adult client the nurse should A. use leading questions for valid responses. B. provide the client with information as questions arise. C. read each question carefully from the history form. D. complete the interview as quickly as possible.,

B. provide the client with information as questions arise.

During an interview with an adult client for the first time the nurse can clarify the client's statements by A. offering a "laundry list" of descriptors. B. rephrasing the client's statements. C. repeating verbatim what the client has said. D. inferring what the client's statements mean.

B. rephrasing the client's statements.

The nurse observes dimpling in an adult female client's breasts. The nurse should explain to the client that dimpling of the breast may indicate a A. fibroadenoma B. tumor. C. genetic deviation. D. fibrocystic breast.

B. tumor.

An adult client tells the nurse that his father had cataracts. He asks the nurse about risk factors for cataracts. The nurse should instruct the client that a potential risk factor is A. lack of vitamin C in the diet. B. ultraviolet light exposure. C. obesity. D. use of antibiotics.

B. ultraviolet light exposure.

The nurse is caring for a client who is having nothing by mouth (NPO) on the first postoperative day. The client's blood pressure was 120/80 mm Hg approximately 4 hours ago but it is now 140/88 mm Hg. The nurse should ask the client which of the following questions? A. "Are you taking any medications for hypertension?" B. "Do you have enough blankets to stay warm?" C. "Are you having pain from your surgery?" D. "What is your typical blood pressure reading?"

C. "Are you having pain from your surgery?"

Waist circumference guidelines may not be accurate for adult clients who are shorter than five feet in height. This restriction is also a concern for which other anthropometric measurement? A. Ideal weight B. Mid-arm circumference C. Body mass index D. Triceps skin-fold measurements

C. Body mass index

The nurse is caring for a female client during her first postoperative day after a temporary colostomy. The client refuses to look at the colostomy bag or the area. A priority nursing diagnosis for this client is A. Denial related to temporary colostomy. B. Fear related to potential outcome of surgery. C. Disturbed body image related to temporary colostomy. D. Altered role functioning related to frequent colostomy bag changes.

C. Disturbed body image related to temporary colostomy.

The nurse is planning a presentation on osteoporosis to a group of high school students. Which of the following should the nurse plan to include in the presentation? A. Bone density rises to a peak at age 50 for both sexes. B. Bone density in the Asian population is higher than in the white population. C. Moderate strenuous exercise tends to increase bone density. D. Approxinately 5 million fractures in the United States are due to osteoporosis.

C. Moderate strenuous exercise tends to increase bone density.

While assessing the nails of an adult client the nurse observes Beau's lines. The norsq should ask the client if he has had A. chemotherapy. B. radiation. C. a recent illness. D. steroid therapy.,

C. a recent illness.

While assessing muscle strength in an older adult client the nurse determines that the client's knee joint has a rating of 3 and exhibits active motion against gravity. The nurse should document the client's muscle strength as beinglhaving A. normal. B. slight weakness. C. average weakness. D. poor range of motion.

C. average weakness.

While examining a client the nurse plans to palpate temperature of the skin by using the A. fingertips of the hand. B. ulnar surface of the hand. C. dorsal surface of the hand. D. palmar surface of the hand.

C. dorsal surface of the hand

While assessing the plantar reflex of an adslt client the nurse observes a positive Babinsk~r eflex. The nurse suspects that the client may be exhibiting signs of A. meningeal irritation. B. diabetes mellitus. C. drug intoxication. D. lower motor neuron lesions.

C. drug intoxication.

The cone of light is located in the A. inner ear. B. middle ear. C. external ear. D. semicircular canal.

C. external ear.

The bony orbit and far cushion of the eye serves as a A. caruncle. B. channel. C. protector. D. filter.

C. protector.

The nurse is preparing to measure the triceps skin-fold of an adult client. The nurse should A. ask the client to assume a sitting position. B. measure the triceps skin-fold in the dominant arm. C. repeat the procedure three times and average the measurements. D. pull the skin toward the muscle mass of the arm.

C. repeat the procedure three times and average the measurements.

The skin folds of the labia rnajora are composed of adipose tissue sebaceous glands, and A. Skene's ducts. B. vestibular glands. C. sweat glands. D. Bartholin's glands.,

C. sweat glands.

After assessing the breasts of a female client the nurse should explain to the client that most breast tumors occur in the A. upper inner quadrant. B. lower inner quadrant. C. upper outer quadrant. D. lower outer quadrant.,

C. upper outer quadrant.

During the interview between a nurse and a client the nurse and the client collaborate to identify problems and goals. This occurs during the phase of the interview termed A. introductory. B. ongoing. C. working. D. closure.,

C. working.

The diencephalon of the brain consists of the A. pons and brainstem. B. medulla oblongata and cerebrum. C. cerebellum and midbrain. D. thalamus and hypothalamus.,

D, thalamus and hypothalamus.

The nurse is preparing ro perform a musculoskeletal examination on an adult client. The nurse has explained the examination procedure to the client. The nurse determines that the client needs further instructions when the client says A. "You will be asking me to change positions often." B. "You'll be comparing bilateral joints." C. "You'll be assessing the size and strength of my joints." D. "You'll continue with range of motion even if I have discomfort."

D. "You'll continue with range of motion even if I have discomfort."

Which of the following statements about systems is not true? A. A system is an interacting whole formed of many parts. B. A system is more complex than the individual parts. C. Systems interact with one another constantly. D. All nursing conceptual frameworks consider the interacting systems of client and context.

D. All nursing conceptual frameworks consider the interacting systems of client and context.

Fibrous tissue that provides support for the glandular tissue of the breasts is termed A. lateral ligaments. B. Wharton's ligaments. C. pectoral ligaments. D. Cooper's ligaments.

D. Cooper's ligaments.

A client has had a recent mastectomy and visits the clinic for postoperative evaluation. The client tells the nurse that she has been depressed and feels as if she is no longer a woman. The most appropriate nursing diagnosis for this client is A. Ineffective individual coping related to mastectomy. B. Fear of additional breast cancer related to presence of risk factors. C. PC: Hematoma after mastectomy. D. Disturbed body image related to mastectomy.

D. Disturbed body image related to mastectomy.

The nurse is examining an adult client's range of motion in the shoulders. The client is unable to shrug her shoulders against resistance. The nurse suspects that the client has a lesion of cranial nerve A. VIII. B. IX. C. X. D. XI.

D. XI.

Elevated sebaceous glands known as Montgomery's glands, are located in the breast's A. nipples. B. hair follicles. C. lactiferous ducts. D. areolas.,

D. areolas.

An adult client visits the clinic and tells the nurse that she suspects she has a urinary tract infection. To detect tenderness over the client's kidneys the nurse should instruct the client that the nurse will be performing A. moderate palpation. B. deep palpation. C. indirect percussion. D. blunt percussion.,

D. blunt percussion.

The nurse assesses an adult client's breath sounds and hears sonorous wheezes primarily during the client's expiration. The nurse should refer the client to a physician for possible A. asthma. B. chronic emphysema. C. pleuritis. D. bronchitis.,

D. bronchitis.

The conjunctiva of the eye is divided into the palpebral portion and the A. canthus portion. B. intraocular portion. C. nasolacrimal portion. D. bulbar portion.

D. bulbar portion.

The lining of tlie trachea and bronchi which serves to remove dust, foreign bodies, and bacteria, is termed the A. bronchioles. B. alveolar sacs. C. alveolar ducts. D. cilia.,

D. cilia.

The nurse plans to assess an adult client's kidneys for tenderness. The nurse should assess the area at the A. right upper quadrant B. left upper quadrant. C. external oblique angle. D. costovertebral angle.

D. costovertebral angle.

The nurse is interviewing a client in the clinic for the first time. When the client tells the nurse that he smokes "about two packs of cigarettes a day " the nurse should A. look at the client with a frown. B. tell the client that he is spending a lot of money foolishly. C. provide the client with a list of dangers associated with smoking. D. encourage the client to quit smoking.,

D. encourage the client to quit smoking.

The outer layer of the vaginal wall is under the direct influence of A. androgen. B. progesterone. C. aldosterone. D. estrogen.

D. estrogen.

Common signs or symptoms of disease in the oldest-old include all of the below except A. weakness. B. confusion. C. falls. D. fever.

D. fever.

An adult male client visits the clinic and tells the nurse that he believes he has athlete's foot. The nurse observes that the client has linear cracks in the skin on both feet. The nurse should document the presence of A. ulcers. B. erosion. C. scales. D. fissures.

D. fissures.

The visible portion of the clitoris is termed the A. corpus. B. crura. C. vestibule. D. glans.

D. glans.

The cranial nerve that has sensory fibers for taste and fibers that result in the "gag reflex" is the A. vagus. B. hypoglossal. C. trigeminal. D. glossopharyngeal.

D. glossopharyngeal.

The nurse is assessing an adult client with a diagnosis of sinus arrhythmia. The nurse should explain to the client that this indicates that the A. heartbeats are followed by a pause. B. ventricular contraction occurs irregularly. C. S1 and S2 sounds are both split. D. heart rate speeds up and slows down during a cycle.,

D. heart rate speeds up and slows down during a cycle.

Articulation between the head of the femur and the acetabulum is in the A. knee joint. B. tibial joint. C. ankle joint. D. hip joint.

D. hip joint.

During a comprehensive assessment the primary technique used by the nurse throughout the examination is A. palpation. B. percussion. C. auscultation. D. inspection.,

D. inspection.

X benign skin lesion con~monlys een in the aged is A. squalnous cell carcinoma. B. shingles. C. actinic keratosis. D. lentigenes.

D. lentigenes.

A medical examination differs from a comprehensive nursing examination in that the medical examination focuses primarily on the client's A. physiologic status. B. holistic wellness status. C. developmental history. D. level of functioning.

A. Physiological status

Which of the following is the best reason for the nurse to assess the client's community? A. There is an interaction of culture family, and envrironmental systems with the client's emotional and physical systems affecting the client's health status. B. Pollution from an unhealthy community can affect the client's physical health. C. Emotional stress from a work environment can cause both emotional and physical illness. D. Community is the context in which health care professionals care for clients.

A. There is an interaction of culture, family, and envrironmental systems with the client's emotional and physical systems affecting the client's health status.

During a gynecologic examination the nurse observes that the client has a yellow-green frothy vaginal discharge. The nurse should plan to test the client for possible A. Trichomonas uaginalis infection. B. bacterial vaginosis. C. atrophic vaginitis. D. Chlamydia trachomatis infection.

A. Trichomonas uaginalis infection.

The nurse has interviewed a Hispanic client with li~nitedE nglish skills for the first time. The nurse observes that the client is reluctant to reveal personal information and believes in a hot-cold syndro~neo f disease causation. The nurse should A. acknowledge the client's cultural differences. B. request a family member to interpret for the client. C. use slang terms to identify certain body parts. D. remain in a standing position during the interview.

A. acknowledge the client's cultural differences.

A bony ridge located at the point where the ~nanubriurna rticulates with the body of the sternum is termed the sternal A. angle. B. notch. C. space. D. prominens.

A. angle.

The nurse is preparing to assess the cardiovascular system of an adult client with emphysema. The nurse anticipates that there [nay be some difficulty palpating the client's A. apical pulse. B. breath sounds. C. jugular veins. D. carotid arteries.

A. apical pulse.

The nurse is preparing to use a Doppler ultrasound probe to detect blood flow in the femoral artery of an adult client. The nurse should A. apply K-Y jelly to the client's skin. B. place the client in a supine position with the head flat. C. place the tip of the probe in a 30-degree angle to the artery. D. apply gel used for electrocardiography (ECG) to the client's skin.

A. apply K-Y jelly to the client's skin.

The apocrine glands are dormant until puberty and are concentrated in the axillae the perineum, and the A. areola of the breast. B. entire skin surface. C. soles of the feet. D. adipose tissue.,

A. areola of the breast.

Before beginning a comprehensive health assessment of an adult client the nurse should explain to the client that the purpose of the assessment is to A. arrive at conclusions about the client's health. B. document any physical symptoms the client may have. C. contribute to the medical diagnosis. D. validate the data collected.,

A. arrive at conclusions about the client's health.

A client visits the clinic and tells the nurse that she has joint pain in her hands especially in the morning. The nurse should assess the client further for signs and symptoms of A. arthritis. B. osteoporosis. C. carpal tunnel syndrome. D. a neurologic disorder.,

A. arthritis.

While assessing the elbow of an adult client the client complains of pain and swelling. The nurse should further assess the client for A. arthritis. B. ganglion cyst. C. carpal tunnel syndrome. D. nerve damage.,

A. arthritis.

Before beginning the examination of the genitalia of an adult male client the nurse should A. ask the client to empty his bladder. B. tell the client that he will remain in a supine position. C. ask the client to leave his shirt in place. D. tell the client that he may leave his underwear in place.,

A. ask the client to empty his bladder.

The nurse is planning to assess the abdomen of an adult male client. Before the nurse begins the assessment the nurse should A. ask the client to empty his bladder. B. place the client in a side-lying position. C. ask the client to hold his breath for a few seconds. D. tell the client to raise his arms above his head.,

A. ask the client to empty his bladder.

The nurse is planning to auscultate a female adult client's carotid arteries. The nurse should plan to A. ask the client to hold her breath. B. palpate the arteries before auscultation. C. place the diaphragm of the stethoscope over the artery. D. ask the client to breathe normally.

A. ask the client to hold her breath.

The nurse is assessing the neurologic system of an adult client. To test the client's motor function of the facial nerve the nurse should A. ask the client to purse the lips. B. ask the client to open the mouth and say "ah." C. note tlie presence of a gag reflex. D. observe the client s\vallo\v a sip of water.,

A. ask the client to purse the lips.

The nurse is assessing the mouth of an older adult and observes that the client appears to have poorly fitting dentures. The nurse should instruct the client that she may be at greater risk for A. aspiration. B. malocclusion. C. gingivitis. D. throat soreness.

A. aspiration.

The nurse is interviewing a 73-year-old client for the first time. The nurse should first A. assess the client's hearing acuity. B. establish rapport with the client. C. obtain biographic data. D. use medical terminology appropriately.

A. assess the client's hearing acuity.

The semilunar valves are located A. at the exit of each ventricle at the beginning of the great vessels. B. between the right atrium and the right ventricle. C. between the left atrium and the left ventricle. D. at the beginning of the ascending aorta.

A. at the exit of each ventricle at the beginning of the great vessels.

During the client interview the nurse uses nonverbal expressions appropriately when the nurse A. avoids excessive eye contact with the client. B. remains expressionless throughout the interview. C. uses touch in a friendly manner to establish rapport. D. displays mental distancing during the interview.,

A. avoids excessive eye contact with the client.

The nurse is performing a speculum examination on an adult woman. The nurse is having difficulty inserting the speculum because the client is unable to relax. The nurse should ask the client to A. bear down. B. hold her breath. C. use imagery to relax. D. take a deep breath.

A. bear down.

The bicuspid or mitral, valve is located A. between the left atrium and the left ventricle. B. between the right atrium and the right ventricle. C. at the beginning of the ascending aorta. D. at the exit of each ventricle near the great vessels.,

A. between the left atrium and the left ventricle.

A client visits the clinic and tells the nurse that his stools have been pale for the past 2 days and his skin has been itching. The nurse should refer the client to a physician for possible A. biliary disease. B. cancer. C. gastrointestinal infection D. hemorrhoids.

A. biliary disease.

The nurse observes an orange-peel appearance or peau d'orange, of the areolae of a client's breasts. The nurse should explain to the client that this is most likely due to A. blocked lymphatic drainage. B. fibrocystic breast disease. C. fibroadenomas. D. radiation therapy.

A. blocked lymphatic drainage.

While assessing an adult client's feet for fungal disease using a \Vood's light the nurse documents the presence of a fungus when the fluorescence is A. blue. B. red. C. yellow. D. purple.,

A. blue.

While palpating the apex left sternal border, and base in an adult client, the nurse detects a thrill. The nurse should further assess the client for A. cardiac murmur. B. left-sided heart failure. C. constrictive pericarditis. D. congestive heart failure.,

A. cardiac murmur.

The middle layer of the eye is known as the A. choroid layer. B. scleral layer. C. retinal layer. D. optic layer.

A. choroid layer.

The nurse assesses an adult client and observes that the client's breathing pattern is very labored and noisy with occasional coughing. The nurse should refer the client to a physician for possible A. chronic bronchitis. B. atelectasis. C. renal failure. D. congestive heart failure.,

A. chronic bronchitis.

The nurse is preparing to percuss a client's reflexes in his arms. To use the reinforcement technique the nurse should ask the client to A. clench his law. B. stretch the opposite arm. C. hold his neck toward the floor, D. straighten his legs forward.,

A. clench his law.

An adult client visits the clinic and tells the nurse that she has had headaches recently that are intense and stabbing and often occur in the late evening. The nurse should document the presence of A. cluster headaches. B. migraine headaches. C. tension headaches. D. tumor-related headaches.

A. cluster headaches.

The P-wave phase of an electrocardiogram (ECG) represents A. conduction of the impulse throughout the atria. B. conduction of the impulse throughout the ventricles. C. ventricular repolarization. D. ventricular polarization.

A. conduction of the impulse throughout the atria.

Although the assessment phase of the nursing process precedes the other phases the assessment phase is A. continuous. B. compieted on admission. C. linear. D. performed only by nurses.

A. continuous.

The cerebrum is divided into right and left hemispheres which are joined together by the A. corpus callosum. B. diencephalon. C. medulla oblongata. D. pons.,

A. corpus callosum.

An elderly client is seen by the nurse in the neighborhood clinic. The nurse observes that the client is dressed in several layers of clothing although the temperature is warm outside. The nurse suspects that the client's cold intolerance is a result of A. decreased body metabolism. B. neurologic deficits. C. recent surgery. D. pancreatic disease.,

A. decreased body metabolism.

An older adult client visits the clinic complaining of urinary incontinence. The nurse should explain to the client that this is often due to A. decreased urethral elasticity. B. atrophy of the vaginal mucosa. C. change in the vaginal pH. D. decreased estrogen production.

A. decreased urethral elasticity.

The nurse is planning to inspect an adult client's mouth using a tongue depressor. The nurse should plan to A. depress the tongue blade slightly off center. B. depress the tongue blade as close to the center as possible. C. ask the client to keep the mouth partially open. D. insert the tongue blade at the back of the client's tongue.,

A. depress the tongue blade slightly off center.

A client visits the clinic and tells the nurse that he has not been feeling very well. The nurse observes that the client's speech is slow the client has a disheveled appearance, and he maintains poor eye contact with the nurse. The nurse should further assess the client for A. depression. B. delirium. C. hallucinations. D. schizophrenia.

A. depression.

The nurse has assessed the respiratory pattern of an adult client. The nurse determines that the client is exhibiting Kussmaul's respirations with hyperventilation. The nurse should contact the client's physician because this type of respiratory pattern usually indicates A. diabetic ketoacidosis. B. central nervous system injury. C. drug overdose. D. congestive heart failure.

A. diabetic ketoacidosis.

A fter assessing a client in pain the nurse A. documents the exact description given by the client. B. chooses from the list of pain descriptors what best reflects the client's description. C. asks the family to describe how they view the client's pain. D. documents how helshe best sees the clienr's pain.,

A. documents the exact description given by the client.

While assessing the eye of an adult client the nurse observes an inward turning of the client's left eye. The nurse should document the client's A. esotropia. B. strabismus. C. phoria. D. exotropia.,

A. esotropia.

An assessment form commonly used in long-term care facilities is the nursing minimum data set. One primary advantage to this type of assessment form is that it A. establishes comparability of nursing data across clinical populations. B. clusters all the nursing and medical diagnoses in one place. C. allows for individualization for each client in the health care setting. D. uses a flow chart format for easy documentation of objective data.

A. establishes comparability of nursing data across clinical populations.

Bones contain yellow marrow that i s composed mainly of A. fat. B. protein. C. cartilage. D. carbohydrates.

A. fat.

In the hearing pathway hair cells of the spiral organ of Corti are stimulated by movement of A. fluid. B. sound. C. air. D. bone.,

A. fluid.

A client visits the clinic and tells the nurse that she has had "runny diarrhea" for 2 days. The nurse should assess the client for A. gastrointestinal infection. B. fecal impaction. C. constipation. D. hemorrhoids.

A. gastrointestinal infection.

The corpora spongiosum extends distally to form the acorn-shaped A. glans. B. frenulum. C. corona. D. scrotum.

A. glans.

An adult client tells the nurse that her peripheral vision isn't what it used to be and she has a blind spot in her left eye. The nurse should refer the client for evaluation of possible A. glaucoma. B. increased intracranial pressure. C. bacterial infection. D. migraine headaches.

A. glaucoma.

An adult client tells the nurse that he ears sardines every day. The nurse should instruct the client that a diet high in purines can contribute to A. gouty arthritis. B. osteomalacia. C. bone fractures. D. osteomyelitis.

A. gouty arthritis.

The portion of the brain that rims the surfaces of the cerebral hemispheres forming the cerebral cortex is the A. gray matter. B. cerebellum. C. diencephalon. D. brainstem.

A. gray matter.

While assessing the feet of an adult client the nurse notes that the client's great toes are deviated, with overlapping of the second toes. The client states that there is pain on the medial side. The nurse should refer the client to a physician for possible A. hallux valgus. B. pes planus. C. pes cavus. D. verruca vulgaris.,

A. hallux valgus.

While assessing the feet of an adult client the nurse observes hyperextension of the metatarsophalangeal joint with flexion at the proximal interphalangeal joint on the client's second toes. The nurse should refer the client to a physician for possible A. hammer toes. B. gouty arthritis. C. calluses. D. hallux valgus.

A. hammer toes.

The nurse has discussed the risks for breast cancer with a group of high school seniors. The nurse determines that one of the students needs further instructions when the student says that one risk factor is A. having a baby before the age of 20 years. B. a family history of breast cancer. C. consumption of a high-fat diet. D. late menopause.

A. having a baby before the age of 20 years.

The nurse is caring for an adult female client whose body mass index is 38.7. The nurse should instruct the client that she is at greater risk for: A. heart attack. B. osteoporosis. C. rheumatoid arthritis. D. stomach cancer.

A. heart attack.

While examining the spine of an adult client the nurse notes that the client has a flattened lumbar curvature. l'lie nurse should refer the client to a physician for possible A. herniated disc. B. scoliosis. C. kyphosis. D. cervical disc degeneration.,

A. herniated disc.

An older adult client visits the clinic and tells the nurse that she has had shooting pains in both of her legs. The nurse should assess the client for signs and symptoms of A. herniated intervertebral disc. B. rheumatoid arthritis. C. osteoporosis. D. metastases.

A. herniated intervertebral disc

While performing a gynecologic examination the nurse observes small, painful, ulcer-like lesions with red bases on the client's labia. The nurse should refer the client to a physician for possible A. herpes simplex virus infection. B. syphilis. C. lice. D. herpes zoster virus infection.

A. herpes simplex virus infection.

A11 adult client tells the nurse that his father died of a massive coronary at the age of 65. The nurse should explain to the client that one of the risk factors for coronary heart disease is A. high serum level of low-density lipoproteins. B. low carbohydrate diets. C. high serum level of high-density lipoproteins. D. diets that are hig- h in antioxidant vitamins.

A. high serum level of low-density lipoproteins.

During a comprehensive assessment of the lungs of an adult client with a diagnosis of emphysema the nurse anticipates that during percussion the client will exhibit A. hyperresonance. B. tympany. C. dullness. D. flatness.,

A. hyperresonance.

A 20-year-old client visits the outpatient center and tells the nurse that he has been experiencing sudden generalized hair loss. After determining that the client has not received radiation or chemotherapy the nurse should further assess the client for signs and symptoms of A. hypothyroidism. B. hyperthyroidism. C. infectious conditions. D. hypoparathyroidism.,

A. hypothyroidism.

While assessing the nails of an older adult the nurse observes early clubbing. The nurse should further evaluate the client for signs and synlptotns of A. hypoxia. B. trauma. C. anemia. D. infection.,

A. hypoxia.

Because the nurse realizes that spirituality varies information gained will assist the nurse in A. individualizing interventions to meet specific needs. B. diagnosing the client with spiritual distress. C. teaching strict adherence to rituals and practices to improve outcomes. D. providing an overview of widely held beliefs from the major religions,

A. individualizing interventions to meet specific needs.

While assessing the cervix of an adult client the nurse observes a yellowish discharge from the cervix. The nurse should further assess the client for a/an A. infection. B. abnormal lesion. C. positive pregnancy test result. D. polyp.,

A. infection.

While assessing the ears of an adult client the nurse observes that the tympanic membrane is completely immobile. The nurse should further assess the client for signs and symptoms of A. infection. B. skull injury. C. vestibular disorders. D. healed perforations.

A. infection.

The only layer of the skin that undergoes cell division is the A. innermost layer of the epidermis. B. outermost layer of the epidermis. C. innermost layer of the dermis. D. outermost layer of the dermis.

A. innermost layer of the epidermis.

Examination of the skin should be A. integrated throughout the head-to-toe examination. B. completed at the beginning of the physical assessment before proceeding to other parts of the exam. C. performed at the very end of the physical assessment. D. integrated and completed only with the musculoskeleteal examination.

A. integrated throughout the head-to-toe examination.

While assessing the abdominal sounds of an adult client the nurse hears high-pitched tingling sounds throughout the distended abdomen. The nurse should refer the client to a physician for possible A. intestinal obstruction. B. gastroenteritis. C. inflamed appendix. D. cirrhosis of the liver.,

A. intestinal obstruction.

The inguinal canal in a male client is located A. just above and parallel to the inguinal ligament. B. anteriorly above the symphysis pubis. C. anterior to the external inguinal ring. D. posterior to the superior iliac ring.

A. just above and parallel to the inguinal ligament.

The fourth heart sound is an A. low-frequency sound best heard with the bell of the stethoscope. B. abnormal finding in trained athletes. C. sound that can be heard in the absence of atrial contraction. D. sound that may increase during expiration.

A. low-frequency sound best heard with the bell of the stethoscope.

While assessing an adult client the nurse observes freckles on the client's face. The nurse should document the presence of A. macules. B. papules. C. plaques. D. bulla.,

A. macules.

While interviewing a client for the first time the nurse is using a standardized nursing history form. The nurse should A. maintain eye contact while asking the questions from the form. B. read the questions verbatim from the form. C. ask the client to complete the form. D. ask leading questions throughout the interview.,

A. maintain eye contact while asking the questions from the form.

The tarsal plates of the upper eyelid contain A. meibomian glands. B. sebaceous glands.. C. tear ducts. D. ocular muscles.

A. meibomian glands.

While caring for an 80-year-old client In his home the nurse determines that the client's temperature is 96.3°F. The nurse determines that the client is most likely exhibiting A. normal changes that occur with the aging process. B. hypothermia that occurs before an infectious process. C. a metabolic disorder resulting in circulatory changes. D. an immune disorder resulting in low platelet count.,

A. normal changes that occur with the aging process.

The nurse detects paradoxical pulses in an adult client during an examination. The nurse should explain to the client that paradoxical pulses are usually indicative of A. obstructive lung disease. B. left-sided heart failure. C. premature ventricular contractions. D. aortic stenosis.

A. obstructive lung disease

While assessing the pupils of a hospitalized adult client the nurse observes that the client's pupils arc dilated to 6 centinieters. The nurse suspects that the client is exhibiting signs ot A. oculomotor nerve paralysis. B. damage to the pons. C. alcohol abuse. D. cocaine abuse.,

A. oculomotor nerve paralysis

The optic nerves from each eyeball cross at the A. optic chiasma. B. vitreous humor. C. optic disc. D. visual cortex.

A. optic chiasma.

A female client visits the clinic and tells the nurse that she began menarche at the age of 16 years. The nurse should instruct the client that she is at a higher risk for A. osteoporosis. B. osteomyelitis. C. rheumatoid arthritis. D. lordosis.

A. osteoporosis.

Squarnous cell carcinoma is associated with A. overall amount of sun exposure. B. intermittent exposure to ultraviolet rays. C. precursor lesions. D. an increase in the rates of melanoma.

A. overall amount of sun exposure.

l'he tlli~id o~ll~le-layresedr ous membrane that lines the chest cavity is termed A. parietal pleura. B. pulmonary pleura. C. visceral pleura. D. thoracic pleura.

A. parietal pleura

During an interview the nurse collects both subjective and objective data from an adult client. Subjective data would include the client's A. perception of pain. B. height. C. weight. D. temperature.,

A. perception of pain.

The outermost layer of the vaginal wall is composed of A. pink squamous epithelium and connective tissue. B. the vascular supply nerves, and lymphatic channels. C. smooth muscle and connective tissue. D. connective tissue and the vascular network.,

A. pink squamous epithelium and connective tissue.

A client visits the clinic and complains of pain in his knees. The nurse explains that a ballottement test will be performed. To perform the ballottement test the nurse should A. place the left thumb and index finger on either side of the patella. B. use the ball of the hand to firmly stroke the medial side of the knee. C. press the lateral side of the knee and inspect for swelling. D. palpate for tenderness 10 centimeters above the patella.,

A. place the left thumb and index finger on either side of the patella.

While assessing an adult client the nurse observes an elevated, palpable, solid mass with a circumscribed border that measures 1. centimeter. The nurse documents this as a A. plaque. B. macule. C. papule. D. patch.,

A. plaque.

While assessing an adult client's lungs during the postoperative period the nurse detects coarse crackles. The nurse should refer the client to a physician for possible A. pneumonia. B. pleuritis. C. bronchitis. D. asthma.,

A. pneumonia.

While assessing the anus of an adult client the nurse detects the presence of small nodules. The nurse should refer the client to a physician for possible A. polyps. B. anorectal fistula. C. hemorrhoids. D. rectocele.,

A. polyps.

The nurse is preparing to examine an adult client's eyes using a Snellen chart. The nurse should A. position the client 20 feet away from the chart. B. ask the client to remove his glasses. C. ask the client to read each line with both eyes open. D. instruct the client to begin reading from the bottom of the chart.,

A. position the client 20 feet away from the chart.

The nurse is preparing to inspect the nose of an adult client with an otoscope. The nurse plans to A. position the handle of the otoscope to one side. B. tip the client's head as far back as possible. C. direct the otoscope tip quickly back and down the nostril. D. position the handle of the otoscope straight and up.

A. position the handle of the otoscope to one side.

The sinoatrial node of the heart is located on the A. posterior wall of the right atrium. B. anterior wall of the right atrium. C. upper intraventricular system. D. anterior wall of the left atrium.

A. posterior wall of the right atrium.

The anterior chest area that overlies the heart and great vessels is called the A. precordium. B. epicardium. C. myocardium. D. endocardium.

A. precordium.

The nurse is recording admission data for an adult client using a cued or checklist type of assessment form. This type of assessment form A. prevents missed questions during data collection. B. covers all the data that a client may provide. C. clusters the assessment data with nursing diagnoses. D. establishes comparability of data across populations.

A. prevents missed questions during data collection.

At puberty the female breasts enlarge in response to estrogen and A. progesterone. B. aldosterone. C. lactogen. D. prolactin.,

A. progesterone.

While examining the prostate gland of an older adult the nurse detects hard fixed nodules. The nurse should refer the client to a physician for possible A. prostate cancer. B. benign prostatic hypertrophy. C. acute prostatitis. D. prostatocystitis.,

A. prostate cancer.

The nurse assesses an adult client's head and neck. While examining the carotid arteries the nurse assesses each artery iridividually in order to prevent a A. reduction of the blood supply to the brain. B. rapid rise in the client's pulse rate. C. premature ventricular heart sound. D. decreased pulse pressure.,

A. reduction of the blood supply to the brain.

A nurse assesses the mouth of an adlilt male cl~enat nd observes a rough crusty, eroded area. The nurse should A. refer the client for further evaluation. B. document the presence of herpes simplex. C. ask the client if his gums bleed. D. documelit the presence of a canker sore.,

A. refer the client for further evaluation.

A male client tells the nurse that he has received a diagnosis of hernia. He visits the clinic because he is nauseated and has extreme tenderness on the left side. The nurse should A. refer the client to an emergency room. B. try to push the mass into the abdomen. C. assess for a mass on the right side. D. assess the client's vital signs.

A. refer the client to an emergency room.

A female client has scheduled a physical examination including a Pap smear. The nurse should instruct the client to A. refrain from douching 48 hours before the examination. B. bring in a urine sample for testing. C. drink a large volume of fluid before the examination. D. refrain from using talcum powder after her shower.

A. refrain from douching 48 hours before the examination.

While assessing the thoracic area of an adult client the nurse plans to auscultate for voice sounds. To assess bronchophony, the nurse should ask the client to A. repeat the phrase "ninety-nine." B. repeat the letter "E." C. whisper the phrase "one-two-three." D. repeat the letter "A."

A. repeat the phrase "ninety-nine."

The colon originates in this abdominal urea: the A. right lower quadrant. B. right upper quadrant. C. left lower quadrant. D. left upper quadrant.

A. right lower quadrant.

To percuss the liver of an adult client the nurse should begin the abdominal assessment at the client's A. right upper quadrant. B. right lower quadrant. C. left upper quadrant. D. left lower quadrant.

A. right upper quadrant.

While assessing the range of motion in an adult client's shoulders the client expresses pain and exhibits limited abduction and muscle weakness. The nurse plans to refer the client to a physician for possible A. rotator cuff tear. B. nerve damage. C. cervical disc degeneration. D. tendonitis.,

A. rotator cuff tear.

The nurse is caring for a client during the immediate postoperative period after abdominal surgery. While performing a "neuro check" the nurse should assess the client's A. sensation in the extremities. B. deep tendon reflexes. C. ability to speak. D. recent memory.

A. sensation in the extremities.

W hile assessing an older adult client's respirations the nurse can anticipate that the respiratory pattern may exhibit a A. shorter inspiratory phase. B. longer inspiratory phase. C. shorter expiratory phase. D. longer expiratory phase.,

A. shorter inspiratory phase

T he hypothalamus is responsible for regulating A. sleep cycles. B. nerve impulses. C. memory. D. eye reflexes.

A. sleep cycles.

A client visits the clinic because she experienced bright hematemesis yesterday. The nurse should refer the client to a physician because this symptom is indicative of A. stomach ulcers. B. pancreatic cancer. C. decreased gastric motility. D. abdominal tumors.

A. stomach ulcers.

The primary function of the gallbladder is to A. store and excrete bile. B. aid in the digestion of protein. C. produce alkaline mucus. D. produce hormones.

A. store and excrete bile.

An older adult client visits the clinic for a gynecologic examination. The client tells the nurse that she has been told that she has uterine prolapse. The nurse should further assess the client for A. stress incontinence. B. cystocele. C. a retroverted uterus. D. diastasis recti.

A. stress incontinence.

When the nurse moves a client's leg upward the nurse is performing A. supination. B. external rotation. C. eversion. D. internal rotation.,

A. supination.

An adult male client visits the outpatient center and tells the nurse that he has been experiencing patchy hair loss. The nurse should further assess the client for A. symptoms of stress. B. recent radiation therapy. C. pigmentation irregularities. D. allergies to certain foods.

A. symptoms of stress.

A 45-year-old client tells the nurse that he occasionally sees spots in front of his eyes. The nurse should A. tell the client that these often occur with aging. B. refer the client to an ophthalmologist. C. re-examine the client in 2 weeks. D. assess the client for signs of diabetes.

A. tell the client that these often occur with aging.

Skeletal muscles are attached to bones by A. tendons. B. cartilage. C. fibrous connective tissue. D. ligaments.

A. tendons.

Ethnocentrism is defined as A. the belief that one's cultural values are superior to all others. B. a tvorldview that each of us forms based on our own culture. C. the basis of cultural competence. D. a culture-hound syndrome found in many cultural groups.

A. the belief that one's cultural values are superior to all others.

A female client visits the clinic and tells the nurse that she frequently experiences severe recurring headaches that sometimes last for several days and are accompanied by nausea and vomiting. The nurse determines that the type of headache the client is describing is a A. tnigraine headache. B. cluster headache. C. tension headache. D. tumor-related headache.

A. tnigraine headache.

A 45-year-old male client tells the nurse that he has had problems in having an erection for the last couple of weeks but is "doing better now." The nurse should explain to the client that A. transient periods of erectile dysfunction are common. B. impotence in males should be investigated. C. transient impotence may be indicative of prostate enlargement. D. inguinal hernias have been associated with transient impotence.

A. transient periods of erectile dysfunction are common

While assessing an adult client's abdomen the nurse observes that the client's umbilicus is enlarged and everted. The nurse should refer the client to a physician for possible A. umbilical hernia. B. ascites. C. intra-abdominal bleeding. D. pancrearitis.,

A. umbilical hernia.

A male client visits the clinic and tells the nurse that he has had a white discharge from his penis for the past few days. The nurse should refer the client to a physician for possible A. urethritis. B. gonorrhea. C. herpes infection. D. syphilis.

A. urethritis.

The nurse is interviewing a client in the clinic for the first time. The client appears to have a very limited vocabulary. 'The nurse should plan to A. use very basic lay terminology. B. have a family member present during the interview. C. use standard medical terminology. D. show the client pictures of different symptoms such as the "Faces Pain Chart."

A. use very basic lay terminology.

An extension of the soft palate of the mouth which hangs in the posterior midline of the oropbarynx, is the A. uvula. B. frenulum. C. taste buds. D. sublingual fold.

A. uvula.

The nurse is planning to assess a newly admitted adult client. While gathering data ft-om the client the nurse should A. validate all data before documentation of the data. B. document the data after the entire exa~nination process. C. record the nurse's understanding of the client's problem. D. use medical terms that are comn~onlyu sed in health care settings.,

A. validate all data before documentation of the data.

Short pale, and fine hair that is present over much of the body is termed A. vellus. B. dermal. C. lanugo. D. terminal.,

A. vellus.

During a physical examination the nurse detects warm skin and brown pigmentation around an adult client's ankles. The nurse suspects that the client may be experiencing A. venous insufficiency. B. arterial occlusive disease. C. venous ulcers. D. ankle edema.,

A. venous insufficiency.

A female client tells the nurse that she may be experiencing premenstrual syndrome. An appropriate question for the nurse to ask the client is A. "How often are your menstrual periods?" B. "Do you experience mood swings or bloating?" C. "Are you experiencing regular menstrual cycles?" D. "How old were you when you began to menstruate?"

B. "Do you experience mood swings or bloating?"

During a comprehensive assessment of an adult client the nurse can best hear high-pitched sounds by using a stethoscope with a A. 1-inch bell. B. 1 1/2-inch diaphragm. C. 15-inch flexible tubing. D. 1-inch diaphragm.,

B. 1 1/2-inch diaphragm.

The nurse is caring for a healthy adult client with no history of vision problems. The nurse should tell the client that a thorough eye exa~nination is recommended every A. year. B. 2 years. C. 3 years. D. 4 years.

B. 2 years.

A normal pulse pressure range for an adult client is typically A. 20 to 40 mm Hg. B. 30 to 50 mm Hg. C. 40 to 60 mm Hg. D. 60 to 80 mm Hg.

B. 30 to 50 mm Hg.

The nurse documents that a 45-yearold male is 5 feet 10 inches tall and weighs 215 pounds. He tells the nurse that he "has a good appetite but doesn't get much exercise because of his busy work schedule." An appropriate NANDA nursing diagnosis for this client is A. Normal body nutrition related to healthy eating patterns and good appetite. B. Altered nutrition, more than body requirements related to intake greater than calories expended. C. Risk for altered nutrition, more than body requirements related to lack a routine exercise. D. Obesity related to lack of exercise.

B. Altered nutrition, more than body requirements related to intake greater than calories expended.

The nurse is planning a presentation to a group of adults on the topic of cardiovascular accidents. Which of the following should the nurse plan to include in the teaching plan? A. Strokes are the number one cause of death in the United States. B. Smoking and high cholesterol levels are risk factors for CVA. C. Clients who smoke while taking oral contraceptives are not at higher risk D. Postmenopausal women taking estrogen are at greater risk for CVA.

B. Smoking and high cholesterol levels are risk factors for CVA.

While assessing an adult client's jaw the nurse hears a clicking popping sound, and the client expresses pain in the joint. The nurse should further assess the client for A. arthritis. B. TMJ dysfunction C. bruxism. D. previous fracture.,

B. TMJ dysfunction

An objective assessnlent that is frequently indicated when the subjective assessment reveals a history of falling is A. a 24-110~1fro od diary. B. a Get Up and Go test. C. a tonometry exam. D. palpation of the joints for crepitus.

B. a Get Up and Go test.

The best approach to use when perbraling a total physical examination on a client is A. a toe-to-head integrated assessment of body systems. B. a head-to-toe integrated assessment of body systems. C. a total body system approach examining each body system individually. D. any approach that is convenient for you and the client.

B. a head-to-toe integrated assessment of body systems.

While assessing an adult client's skull the nurse observes that the client's skull and facial bones are larger and thicker than usual. The nurse should assess the client for A. parotid gland enlargement. B. acromegaly. C. Paget's disease. D. Cushing's syndrome.,

B. acromegaly.

Acute pain can be differentiated from chronic pain because A. acute pain always scores more on the visual analog scale than chronic pain. B. acute pain is associated with a recent onset of illness or injury with a duration of less than 6 months whereas chronic pain persists longer than 6 months. C. acute pain is not treated and left to subside on its own, whereas chronic pain is referred for treatment. D. acute pain occurs only in persons aged less than 45 years, whereas chronic pain occurs in persons aged 46 or above,

B. acute pain is associated with a recent onset of illness or injury with a duration of less than 6 months, whereas chronic pain persists longer than 6 months.

An adrllt client visits the clinic and tells the nurse that she feels chest pain and pain down her left arm. The nurse should refer the client to a physician for possible A. congestive heart failure. B. angina. C. palpitations. D. acute anxiety reaction.

B. angina.

The nurse is planning to assess an adult client's thyroid gland. The nurse should plan to A. ask the client to raise the chin. B. approach the client posteriorly. C. turn the client's neck slightly backward. D. place the fingers above the cricoid cartilage.

B. approach the client posteriorly

The apex of each lung is located at the A. level of the diaphragm. B. area slightly above the clavicle. C. level of the sixth rib. D. left oblique fissure.

B. area slightly above the clavicle.

The nurse is assessing an adult client's oral cavity for possible oral cancer. The nurse should explain to the client that the most common site of oral cancer is the A. area on top of the tongue. B. area underneath the tongue. C. inside of the cheeks. D. area near the salivary glands.

B. area underneath the tongue.

To assess for anemia in a dark-skinned client the nurse should observe the client's skin for a color that appears A. greenish. B. ashen. C. bluish. D. olive.,

B. ashen.

The nurse is preparing to perform a head and neck assesslnent of an adult client who has immigrated to the United States from Cambodia. The nurse should first A. explain to the client why the assessment is necessary. B. ask the client if touching the head is permissible. C. determine whether the client desires a family member present. D. examine the lymph nodes of the neck before examining the head.

B. ask the client if touching the head is permissible.

The nurse is preparing to document assessment findings in a client's record. The nurse should A. write in complete sentences with few abbreviations. B. avoid slang terms or labels unless they are direct quotes. C. record how the data were collected. D. use the term normal for normal findings.

B. avoid slang terms or labels unless they are direct quotes.

While assessing an adult client the nurse tests the client for Tinel's sign. The nurse should instruct the client that numbness or tingling may indicate A. arthritis. B. carpal tunnel syndrome. C. tenosynovitis. D. crepitus.

B. carpal tunnel syndrome.

While assessing the skin of an older adult client the nurse observes that the client has small yellowish brown patches on her hands. The nurse should instruct the client that these spots are A. signs of an infectious process. B. caused by aging of the skin in older adults. C. precancerous lesions. D. signs of dermatitis.,

B. caused by aging of the skin in older adults.

The nurse is assessing an older adult client when the client tells the nurse that she has experienced transient blind spots for the last few days. The nurse should refer the client to a physician for possible A. vagus nerve damage. B. cerebral vascular accident. C. spinal cord compression. D. Parkinson's disease.

B. cerebral vascular accident.

The transmission of sound waves through the external ear and middle ear is known as A. perceptive hearing. B. conductive hearing. C. external hearing. D. connective hearing.

B. conductive hearing.

A sign of infection in the elder that is more common than fever is A. pain B. confusion. C. diarrhea. D. cough.

B. confusion.

The nurse has tested the near visual acuity of a 4S-year-old client. The nurse explains to the client that the client has impaired near vision and discusses a possible reason for the condition. The nurse determines that the client has understood the instructions when the client says that presbyopia is usually due to A. congenital cataracts. B. decreased accommodation. C. muscle weakness. D. constant misalignment of the eyes.

B. decreased accommodation.

The nurse is assessing an adult male client when the nurse observes gynecomastia in the client. The nurse should ask the client if he is taking any medications for A. inflammation. B. depression. C. infection. D. ulcers.

B. depression.

To alleviate a client's anxiety during a comprehensive assessment the nurse should A. begin with intrusive procedures first to get them completed quickly. B. explain each procedure being performed and the reason for the procedure. C. remain in the examination room while the client changes into a gown. D. ask the client to sign a consent for the physical examination.,

B. explain each procedure being performed and the reason for the procedure.

The tongue is attached to the hyoid bone and styloid process of the temporal bone and is connected to the floor of the mouth by the A. mandible. B. frenulum. C. gums. D. soft palate.

B. frenulum.

The functional part of the breast that allows for milk production consists of tissue termed A. fibrous. B. glandular. C. adipose. D. lactiferous.

B. glandular.

While assessing an adult male client the nurse detects pimple-like lesions on the client's glans. The nurse explains the need for a referral to the client. The nurse determines that the client has understood the instructions when the client says he may have A. venereal warts. B. herpes infection. C. syphilis. D. gonorrhea.,

B. herpes infection.

An adult client visits the clinic and tells the nurse that he has been experiencing double vision for the past few days. The nurse refers the client to a physician for evaluation of possible A. glaucoma. B. increased intracranial pressure. C. hypertension. D. ophthalmic migraine.

B. increased intracranial pressure.

The nurse is assessing the peripheral vascular systm of an older adult client. The client tells the nurse that her legs "seem cold all the time and sometimes feel tingly." The nurse suspects that the client may be experiencing A. varicose veins. B. intermittent claudication. C. edema. D. thrombophlebitis.

B. intermittent claudication.

The nurse is planning to assess the anus and rectum of <In adult male client. The nurse should position the client in a A. right lateral position. B. left lateral position. C. prone position. D. knee-chest position.

B. left lateral position.

The sigmoid colon is located in this area of the abdomen: the A. left upper quadrant. B. left lower quadrant. C. right upper quadrant. D. right lower quadrant.

B. left lower quadrant.

To palpate the spleen of an adult client the nurse should begin the abdominal assessment of the client at the A. left lower quadrant B. left upper quadrant. C. right upper quadrant. D. right lower quadrant.,

B. left upper quadrant.

Bones in synovial joints are joined together by A. cartilage. B. ligaments. C. tendons. D. periosteal tissue.

B. ligaments.

One advantage for an institution to use an integrated cued/cllecklist type of assessment data form is that it A. allows a comprehensive and thorough picture of the client's symptoms. B. may be easily used by different levels of caregivers which enhances communication. C. provides for easy and rapid documentation across clinical settings and populations. D. includes the 11 health care patterns in an easily readable format.,

B. may be easily used by different levels of caregivers, which enhances communication.

A client visits the outpatient center with a complaint of sudden head and neck pain and stiffness. The client's oral temperature is 100°F. The nurse suspects the client is experiencing symptoms of A. migraine headache. B. meningeal irritation. C. trigeminal neuralgia. D. otitis media.

B. meningeal irritation.

The nurse is preparing to assess the respirations of an alert adult client. The nurse should A. explain to the client that he or she will be counting the client's respirations. B. observe for equal bilateral chest expansion of 1 to 2 inches. C. count for 15 seconds and multiply the number by four to obtain the rate. D. ask the client to lie in a supine position which makes counting the respirations easier.,

B. observe for equal bilateral chest expansion of 1 to 2 inches.

An adult white client visits the clinic for the first time. During assessment of the client's skin the nurse should assess for central cyanosis by observing the client's A. nailbeds. B. oral mucosa. C. sclera. D. palms.,

B. oral mucosa.

The nurse is assessing the bowel sounds of an adult client. After listening to each quadrant the nurse determines that bowel sounds are not present. The nurse should refer the client to a physician for possible A. aortic aneurysm. B. paralytic ileus. C. gastroenteritis. D. fluid and electrolyte imbalances.,

B. paralytic ileus.

The nurse assesses an adult client's thoracic area and observes a markedly sunken sternum and adjacent cartilages. The nurse should document the client's A. pectus thorax. B. pectus excavatum. C. pectus carinatum. D. pectus diaphragm.

B. pectus excavatum

The depth and scope of nursing assessment has expanded significantly over the past several decades primarily because of A. the growing elderly population with chronic illness. B. rapid advances in biomedical knowledge and technology. C. an increase in the number of baccalaureate programs in nursing. D. an increase in the number of nurse practitioners.

B. rapid advances in biomedical knowledge and technology.

A client visits the local clinic after experiencing head trauma. The client tells the nurse that he has a consistent blind spot in his right eye. The nurse should A. examine the area of head trauma. B. refer the client to an ophthalmologist. C. assess the client for double vision. D. ask the client if he sees "halos."

B. refer the client to an ophthalmologist.

When a client in considered in context the term context refers to A. . theoretic perspective. B. setting, environment. C. fanlily history. D. chart content.,

B. setting, environment

A female client tells the nurse that she has pain while urinating. Besides obtaining a urinalysis the nurse should assess the client for A. kidney trauma. B. sexually transmitted disease. C. tumors. D. infestation.,

B. sexually transmitted disease.

The subacromial bursae are contained in the A. temporomandibular joint. B. shoulder joint. C. elbow joint. D. wrist joint.

B. shoulder joint.

Loss of connection with one's spiritual support most often leads to A. a new-found sense of liberation. B. spiritual distress. C. improved sense of health and well-being. D. increased adherence to religious practices.

B. spiritual distress.

The nurse assesses an older adult bedridden client in her home. While assessing the client's buttocks the nurse observes that a small area of the skin is broken and resembles an erosion. The nurse should document the client's pressure ulcer as A. stage I. B. stage II. C. stage III. D. stage IV.,

B. stage II.

The testes in the male scrotum are A. joined with thc cjaculatory duct. B. suspended by the spermatic cord. C. able to produce progesterone. D. the location of the vas deferens.

B. suspended by the spermatic cord.

The nurse is preparing to assess the neck of an adult client. To inspect movement of the client's thyroid gland the nurse should ask the client to A. inhale deeply. B. swallow a small sip of water. C. cough deeply. D. flex the neck to each side.,

B. swallow a small sip of water.

While inspecting the genitalia of a male client the nurse observes a chancre lesion under the foreskin. The nurse has explained this observation to the client. The nurse determines that the client understands the need for a referral when the client says that chancre lesions are associated with A. herpesvirus. B. syphilis C. papillomavirus. D. gonorrhea.,

B. syphilis

All of the following are accurate signs of dehydration in the frail elder except A. a furrowed tongue. B. tenting of the skin when pinched. C. dry warm skin. D. sunken eyes.

B. tenting of the skin when pinched.

The nurse is auscultating the heart sounds of an adult client. To auscultate Erb's point the nurse should place the stethoscope at the A. second intercostal space at the right sternal border. B. third to fifth intercostal space at thc left stcrnal border. C. apex of the heart near the midclavicular line. D. fourth or fifth intercostal space at the left lower sternal border.,

B. third to fifth intercostal space at thc left stcrnal border.

The nurse is planning to test position sensation in an adult female client. To perform this procedure the nurse asks the client to close her eyes while the nurse moves tlie client's A. arm away from the body. B. toes up or down. C. hand forward and then backward. D. leg away from the body.,

B. toes up or down.

A client's skin color depends on melanin and carotene contained in the skin and the A. client's genetic background. B. volume of blood circulating in the dermis. C. number of lymph vessels near the dermis. D. vascularity of the apocrine glands.,

B. volume of blood circulating in the dermis.

An adult client tells the nurse that his 80-year-old father is almost completely deaf. After an explanation to the client about risk factors for hearing loss the nurse determines that the client needs further instruction when the client says A. "There is a genetic predisposition to hearing loss." B. "Certain cultural groups have a higher rate of hearing loss." C. "It is difficult to prevent hearing loss or worsening of hearing." D. "Chronic otitis media has been associated with hearing loss.",

C. "It is difficult to prevent hearing loss or worsening of hearing."

A 25-year-old client asks the nurse how often he should have a testicular examination. After instructing the client about the American Cancer Society's guidelines the nurse determines that the client has understood the instructions when he says he should have a testicular examination every A. year. B. 2 years. C. 3 years. D. 4 years.,

C. 3 years.

Blood from the lower trunk and legs drains upward into the inferior vena cava. The percentage of the body's blood volume that is contained in the veins is nearly A. 50%. B. 60%. C. 70%. D. 80%.

C. 70%.

The nurse is planning a presentatio~lo n the topic of colorectal cancer to a group of older adults. Which of the following should the nurse plan to include in the presentation? A. Colorectal cancer rates have steadily fallen over the past 30 years. B. Eighty percent of those diagnosed with colorectal cancer are younger than 50 years of age. C. Diets high in fat and low in fiber are associated with colorectal cancer. D. Colorectal cancer rates are decreasing outside the United States.

C. Diets high in fat and low in fiber are associated with colorectal cancer.

While assessing an adult client's skull the nurse observes that the client's skull bones are acorn shaped and enlarged. The nurse should refer the client to a physician for possible A. Cushing's syndrome. B. scleroderma. C. Paget's disease. D. Parkinson's disease.

C. Paget's disease.

The spinous process termed the vertebra prominens is which cervical vertebra? A. Fifth B. Sixth C. Seventh D. Eighth

C. Seventh

The submandibular glands open under the tongue through openings called A. parasinal ducts. B. Stensen's ducts. C. Wharton's ducts. D. lacrimonasal ducts.

C. Wharton's ducts.

While assessing the anal area of an adult client the nurse detects redness and excoriation. The nurse determines that this sign is most likely due to A. internal hemorrhoids. B. an anorectal fistula. C. a fungal infection. D. previous surgery.,

C. a fungal infection.

A characteristic sign of delirium is A. a significant decline in memory. B. a chronic low mood. C. a rapid decline in level of alertness. D. disorientation to self.

C. a rapid decline in level of alertness.

The abdominal contents are enclosed externally by the abdominal wall musculature-three layers of muscle extending from the back around the flanks, to the front. The outer muscle layer is the external A. rectal abdominis. B. transverse abdominis. C. abdominal oblique. D. umbilical oblique.,

C. abdominal oblique.

Before beginning a physical assessment it is important for the nurse to A. explain to the client in detail how each body system will be assessed. B. explain to the client the purpose of every physical assessment technique you will be using. C. acquire your client's verbal permission to perform the physical examination. D. acquire your client's written pernlission to perfor111 the physical examination.

C. acquire your client's verbal permission to perform the physical examination.

The physical declines of aging often first become noticeable when A. approxin~ately5 0% of function is lost. B. the person is at least 75 years old. C. acute or chronic illness places excessive demands on the body. D. cognitive declines become significant.

C. acute or chronic illness places excessive demands on the body.

The nurse is preparing to examine the breasts of a female client who had a left radical mastectomy 3 years ago. When examining the client the nurse observes redness at the scar area. The nurse should explain to the client that this may be indicative of A. additional tumors. B. poor lymphatic drainage. C. an infectious process. D. metastasis to the right breast.,

C. an infectious process.

During a cardiac examination the nurse can best hear the S, heart sound by placing the stethoscope at the client's A. base of the heart. B. pulmonic valve area. C. apex of the heart. D. second left interspace.,

C. apex of the heart.

The nurse is assessing an older adult client who has lost 5 pounds since her last visit 1 year ago. The client tells the nurse that her husband died 2 months ago. The nurse should further assess the client for A. peptic ulcer. B. bulimia. C. appetite changes. D. pancreatic disorders.

C. appetite changes.

The nurse is preparing to auscultate the posterior thorax of an adult female client. The nurse should A. place the bell of the stethoscope firmly on the posterior chest wall. B. auscultate from the base of the lungs to the apices. C. ask the client to breathe deeply through her mouth. D. ask the client to breathe normally through her nose.

C. ask the client to breathe deeply through her mouth.

While assessing an older adult client the nurse detects a bruit over the carotid artery. The nurse should explain to the client that a bruit is A. a normal sound heard in adult clients. B. a wheezing sound. C. associated with occlusive arterial disease. D. heard when the artery is almost totally occluded.,

C. associated with occlusive arterial disease.

The nurse is preparing to meet a client in the clinic for the first time. After reviewing the client's record the nurse should A. analyze data that have already been collected. B. review any past collaborative problems. C. avoid premature judgments about the client. D. consult with the client's family members.,

C. avoid premature judgments about the client.

When assessing the client for pain the nurse should A. doubt the client when helshe describes the pain. B. assess for underlying causes of pain, then believe the client. C. believe the client when he/she clainls to be in pain. D. assess for the presence of physiologic indicators (such as diaphoresis, tachycardia, etc.), then believe the client.

C. believe the client when he/she clainls to be in pain.

After palpating the radial pulse of an adult client the nurse suspects arterial insufficiency. The nurse should next assess the client's A. femoral pulse. B. popliteal pulse. C. brachial pulse. D. tibial pulse.,

C. brachial pulse.

While assessing an older adult client the client complains of chronic pain and severe limitation of all shoulder movements. The nurse should refer the client to a physician for possible A. rotator cuff tendonitis. B. rheumatoid arthritis. C. calcified tendinitis. D. chronic bursitis.

C. calcified tendinitis.

An adult client tells the nurse that he has been experiencing gradual vision loss. The nurse should A. ask about the client's diet. B. determine whether there is a history of glaucoma. C. check the client's blood pressure. D. ask the client if he has any known allergies.

C. check the client's blood pressure.

The nurse is assessing an African-American client's skin. After the assessment the nurse should instruct the client that African-American persons are more susceptible to A. skin cancers than persons of European origin. B. melanomas if they reside in areas without ozone depletion. C. chronic discoid lupus erythematosus. D. genetic predisposition to melanomas.,

C. chronic discoid lupus erythematosus.

The nurse is planning to perform the Trendelenburg test 011 an adult client. The nurse should explain to the client that this test is used to determine the A. degree of arterial occlusion that exists. B. pulse of a client with poor elasticity. C. competence of the saphenous vein valves. D. severity of thrombophlebitis.

C. competence of the saphenous vein valves.

The nurse has performed the Rinne test on an older adult client. After the test the client reports that her bone conduction sound was heard longer than the air corlduction sound. The nurse determines that the client is most likely experiencing A. normal hearing. B. sensorineural hearing loss. C. conductive hearing loss. D. central hearing loss.

C. conductive hearing loss.

The nurse has assessed a male client and determines that one of the testes is absent. The nurse should explain to the client that this condition is termed A. hypospadias. B. hematocele. C. cryptorchidism. D. orchitis.

C. cryptorchidism.

During palpation of the client's organs the nurse palpates the spleen by applying pressure between 2.5 and 5 centimeters. The nurse is performing A. light palpation. B. moderate palpation. C. deep palpation. D. very deep palpation.,

C. deep palpation.

One of the body's normal physiologic responses to pain is A. hypotension. B. pulse rate below 50/min. C. diaphoresis. D. hypoglycemia.

C. diaphoresis.

A gastrointestinal problenl that often requires emergency treatment in the frail elder is A. lactose intolerance. B. hiatal herilia C. diverticulitis. D. Crohn's disease.

C. diverticulitis.

The nurse is assessing the genitalia of an adult male client when he tells the nurse that his testes are swollen and painful. The nurse should refer the client to a physician for possible A. cancer. B. hydrocele. C. epididyrnitis. D. hematocele.

C. epididyrnitis.

A temale client tells the nurse that her breasts become lumpy and sore before menstruation but get better at the end of the n~enstrualc ycle. The nurse should explain to the client that these symptoms are often associated with A. malignant tumors. B. fibroadenoma. C. fibrocystic breast disease. D. increased estrogen production.

C. fibrocystic breast disease.

The nurse is preparing to palpate the epitrochlear lymph nodes of an adult male client. The nurse should instruct the client to A. assume a supine position. B. rest his arm on the examination table. C. flex his elbow about 90 degrees. D. make a fist with his left hand.

C. flex his elbow about 90 degrees.

Signs of arterial insufficiency in the very old include all of the following except A. paleness of the leg when elevated. B. dusky or mottled appearance of the leg in a dependent position. C. hair loss on the skin. D. cool thin, shiny skin.,

C. hair loss on the skin.

K female client visits the clinic and tells the nurse that she wants to "stay healthy." The nurse observes that the client has diffuse neck enlargement is perspiring, and is quite fidgety. The client tells the nurse that she is "hungry all the time, but I have lost weight." h priority nursing diagnosis for the client is A. imbalanced nutrition: less than body requirements related to energy level. B. ineffective health maintenance related to increased metabolism and hunger. C. health-seeking behaviors related to verbalization of wanting to stay healthy. D. thyroid dysfunction related to neck swelling, perspiration, and fidgeting.

C. health-seeking behaviors related to verbalization of wanting to stay healthy.

The most commonly used method of percussion is A. direct percussion. B. mild percussion. C. indirect percussion. D. blunt percussion.

C. indirect percussion.

A male client tells the nurse that his occupation requires heavy lifting and a great deal of strenuous activity. The nurse should assess the client for A. signs and symptoms of prostate enlargement. B. erectile dysfunction. C. inguinal hernia. D. urinary tract infection.

C. inguinal hernia.

The nails located on the distal phalanges of the fingers and toes, are composed of A. ectodermal cells. B. endodermal cells. C. keratinized epidermal cells. D. stratum cells.,

C. keratinized epidermal cells

For a nurse ro be therapeutic with clients when dealing with sensitive issues such as terminal illness or sexuality the nurse should have A. advanced preparation in this area. B. experience in dealing with these types of clients. C. knowledge of his or her own thoughts and feelings about these issues. D. personal experiences with death, dying, and sexuality.,

C. knowledge of his or her own thoughts and feelings about these issues.

The nurse is preparing to assess an adult client in the clinic. The nurse observes that the client is wearing lightweight clothing although the temperature is below freezing outside. The nurse anticipates that the client may be A. abusing drugs. B. a victim of abuse. C. lacking adequate finances. D. anxious.

C. lacking adequate finances.

The nurse is preparing to examine the sinuses of an adult client. After examining the frontal sinuses the nurse should proceed to examine the A. ethmoidal sinuses. B. laryngeal sinuses. C. maxillary sinuses. D. sphenoidal sinuses.,

C. maxillary sinuses.

While assessing an adult client the nurse detects opening snaps early in diastole during auscultation of the heart. The nurse should refer the client to a physician because this is usually indicative of A. pulmonary hypertension. B. aortic stenosis. C. mitral valve stenosis. D. puln~onary hypotension.,

C. mitral valve stenosis.

While assessing an older adult client the nurse notes decreased range of motion and crepitation as the client tries to bend his knees to his chest. The nurse determines that the client is most likely experiencing A. flexion contractures. B. signs of aging. C. osteoarthritis. D. genu valgum.,

C. osteoarthritis.

A client with insulin-dependent diabetes visits the clinic and complains of painful hip joints. The nurse should assess the client carefully for signs and symptoms of A. arthritis. B. gait difficulties. C. osteomyelitis. D. scoliosis.

C. osteomyelitis.

Cultural beliefs about the causes of breast cancer do not always agree with medical findings. Hispanic Americans often associate breast cancer with A. improper diet. B. punishment from God. C. physical stress. D. evil thoughts.

C. physical stress.

While assessi~lgth e anal area of an adult client the nurse observes a reddened swollen area covered by a small tuft of hair located midline on the lower sacrum. The nurse should refer the client to a physician for possible A. perianal abscess. B. neurologic disorder. C. pilonidal cyst. D. anorectal fistula.

C. pilonidal cyst.

While assessing the feet of an adult client the nurse observes tiny dark spots under a painful callus on the client's foot. The nurse should document the presence of A. corns. B. bunions. C. plantar warts. D. gouty arthritis.

C. plantar warts.

Diminished vibratory sensations and slowed motor responses in advanced age result in A. stiffness and rigidity. B. paresthesia. C. postural instability. D. tremors.

C. postural instability.

One of the functions of a bone i s to A. store fat. B. produce secretions. C. produce blood cells. D. store protein.

C. produce blood cells.

Knowledge of the client's beliefs in the cause of illness can be useful to the nurse in order to A. encourage new beliefs. B. dispel religious teachings if they conflict with the nurse's belief system. C. promote harmony between health and spirituality. D. raise doubt and point out flaws in one's faith.

C. promote harmony between health and spirituality.

To assess an adult client for possible appendicitis and a positive psoas sign the nurse should A. rotate the client's knee internally. B. palpate at the lower right quadrant. C. raise the client's right leg from the hip. D. support the client's right knee and ankle.,

C. raise the client's right leg from the hip.

The nurse is assessing a SO-year-old client's breasts and observes a spontaneous discharge of fluid from the left nipple. The nurse should A. document this as a normal finding. B. ask the client if she has had retracted nipples. C. refer the client for a cytology examination. D. determine whether the client wears a supportive bra.

C. refer the client for a cytology examination

While assessing an older adult client's neck the nurse observes that the client's trachea is pulled to the left side. The nurse should A. ask the client to flex his neck to the left side. B. observe whether the client has difficulty swallowing water. C. refer the client to a physician for further evaluation. D. palpate the cricoid cartilage for smoothness.,

C. refer the client to a physician for further evaluation.

One disadvantage of the open-ended assessment form is that it A. doesn't allow for individualization. B. asks standardized questions. C. requires a lot of time to complete. D. doesn't provide a total picture of the client.

C. requires a lot of time to complete.

The nurse is examining an adult client's eyes. The nurse has explained the positions test to the cl~entT. he nurse determines that the client needs further instructions when the client says that the positions test A. assesses the muscle strength of the eye. B. assesses the functioning of the cranial nerves innervating the eye muscles. C. requires the coverlng of each eye separately. D. requires the cl~entto focus on In object

C. requires the coverlng of each eye separately.

Photoreceptors of the eye are located in the eye's A. ciliary body. B. lens. C. retina. D. pupil.

C. retina.

To prepare for the assessment of a client visiting a neighborhood health care clinic the nurse should first A. discuss the client's symptoms with other team members. B. plan for potential laboratory procedures. C. review the client's health care record. D. determine potential health care resources.,

C. review the client's health care record.

Sensations of temperature pain, and crude and light touch are carried by way of the A. extrapyramidal tract. B. corticospinal tract. C. spinothalamic tract. D. posterior tract.

C. spinothalamic tract.

The nurse is preparing to perform the Rosiberg test on an adult male cllent. The nurse should instruct the cllent to A. squat down as far as he is able to do so. B. keep his eyes open while he bends at the knees. C. stand erect with arms at the sides and feet together. D. touch the tip of his nose with his finger.

C. stand erect with arms at the sides and feet together.

An adult client visits the clinic and complains of tinnitus. The nurse should ask the client if she has been A. dizzy. B. hypotensive. C. taking antibiotics. D. experiencing ear drainage.

C. taking antibiotics.

A key area to assess in older adults with chronic respiratory or cardiac problems and some constant degree of dyspnea is A. nutritional deficiency. B. dysphagia. C. the degree to which dyspnea affects daily function. D. a possible history of in~munosuppression.

C. the degree to which dyspnea affects daily function.

An adult client visits the clinic complaining of a sore throat. After assessing the throat the nurse documents the client's tonsils as 4+. The nurse should explain to llle client that 4+ tonsils are present when the nurse observes tonsils that are A. touching the uvula. B. visible upon inspection. C. touching each other. D. midway between the tonsillar pillars and uvula.,

C. touching each other.

During a physical examination of an adult client the nurse is preparing to auscultate the client's abdomen. The nurse should A. palpate the abdomen before auscultation. B. listen in each quadrant for 15 seconds. C. use the diaphragm of the stethoscope. D. begin auscultation in the left upper quadrant.,

C. use the diaphragm of the stethoscope.

The nurse is preparing to examine the skin of an adult client with a diagnosis of herpes simplex. The nurse plans to measure the client's syniptomatic lesions and measure the size of the client's A. nodules. B. bullae. C. vesicles. D. wheals.

C. vesicles.

The nurse is assessing an elderly postsurgical client in the home. To begin the physical examination the nurse should first assess the client's A. height and weight. B. ability to swallow. C. vital signs. D. gait.,

C. vital signs.

The nurse is preparing to perform a speculum examination on an adult woman. To lubricate the speculum before insertion the nurse should use A. sterile water. B. K-Y jelly. C. warm tap water. D. petroleum jelly.,

C. warm tap water.

To cornpe~isatef ur a stooped poslure atid less flexible knee hip, and shoulder joints, the elderly person often walks A. with a waddling type of gait. B. with one leg slightly dragging behind the other. C. with the feet farther apart and the knees slightly bent. D. with a slight swaying side-to-side motion.,

C. with the feet farther apart and the knees slightly bent.

The nurse is assessing the neurologic system of an adult client. To test the client's use of memory to learn new information the nurse should ask the client A. "What did you have for breakfast?" B. "How old were you when you began working?" C. "Can you repeat rose, hose, nose, clothes?" D. "Can you repeat brown, chair, textbook, tomato?"

D. "Can you repeat brown, chair, textbook, tomato?"

While interviewing an adult client about the client's stress levels and coping responses an appropriate question by the nurse is A. "Do you feel stress at work?" B. "How often do you feel stressed?" C. "Is stress a problem in your life?" D. "How do you manage your stress?",

D. "How do you manage your stress?"

Suzie is a 16-yearold daughter in the Hanes family. She is the youngest of five children. She has had a series of illnesses and does not seem to be regaining her strength. She likes school but is falling behind a bit. Her mother is very attentive to her needs but does not seem overly concerned with the continuing pattern of illness. Which of the followi~lgis most likely a Hanes family belief? A. Fathers are not involved with their children. B. Education is highly valued for sons and daughters. C. Emotional stress from a work environl-[lent can cause both emotional and physical illness. D. Community is the context in which health care professionals care for clients.

D. Community is the context in which health care professionals care for clients.

The nurse is planning a presentation about coronary heart disease for a group of middle-aged adults. Which of the following should be included in the nurse's teaching plan? A. Hispanic clients have a higher incidence of CHD than black or white Americans. B. The incidence of hypertension in the white popi~lationo f the IJnited States is greater than in the black population. C. Women are more likely to have serious stenosis after a heart attack. D. Estrogen replacement therapy in postmenopausal women decreases the risk of heart attack.

D. Estrogen replacement therapy in postmenopausal women decreases the risk of heart attack.

A client who is an active outdoor swimmer recently received a diagnosis of discoid systemic lupus erythematosus. The client visits the clinic for a routine examination and tells the nurse that she continues to swim in the sunlight three times per week. She has accepted her patchy hair loss and wears a wig on occasion. A priority nursing diagnosis for the client is A. Ineffective individual coping related to changes in appearance. B. Anxiety related to loss of outdoor activities and altered skin appearance. C. Dry flaking skin and dull dry hair as a result of disease. D. Risk for ineffective health maintenance related to deficient knowledge of effects of sunlight on skin lesions.

D. Risk for ineffective health maintenance related to deficient knowledge of effects of sunlight on skin lesions.

The nurse is planning a presentation to a group of high school students on the topic of lung cancer. Which of the followillg should the nurse plan to include in the presentation? A. Compared with whites in the United States African Americans have a lower incidence of lung cancer. B. Lung cancer is the third leading cause of death in the United States. C. There is a higher incidence of lui~gc ancer in women than men in the United States. D. Studies have indicated a genetic component in the development of lung cancer.,

D. Studies have indicated a genetic component in the development of lung cancer.

An adult client visits the outpatient clinic and tells the nurse that he has a throbbing aching pain in his right eye. The nurse should assess the client for A. recent exposure to irritants. B. increased intracranial pressure. C. excessive tearing. D. a foreign body in the eye.

D. a foreign body in the eye.

When the nurse moves the client's arm away from the midline of the body the nurse is performing A. adduction. B. external rotation. C. retraction. D. abduction.,

D. abduction.

A 60-year-old male client asks the nurse about risk factors for prostate cancer. The nurse should explain to the client that one possible risk factor is A. a high-carbohydrate diet. B. exposure to sulfur. C. genetic inheritance. D. advanced age.

D. advanced age.

An adult client tells the nurse that she frequently experiences burning and itching of both eyes. The nurse should assess the client far A. a foreign body. B. recent trauma. C. blind spots. D. allergies.

D. allergies.

The nurse has assessed the nose of an adult client and has explained to the client about her thick yellowish nasal discharge. The nurse determines that the client understands the instructions when the client says that the yellowish discharge is 113ost likely due to A. too much smoking. B. chronic allergies. C. trauma to the nasal passages. D. an upper respiratory infection.

D. an upper respiratory infection.

The posterior tibia1 pulse can be palpated at the A. great toe. B. knee. C. top of the foot. D. ankle.

D. ankle.

An adult client visits the clirlic complaining of recurrent ulcers in the mouth. The nurse assesses the client's mouth and observes a painful ulcer. The nurse should document the presence of A. a cancerous lesion. B. Candida albicarzs infection C. an oral ulceration. D. aplithous stomatitis.

D. aplithous stomatitis.

While assessing the peripheral vascular system of an adult client the nurse detects cold clammy skin and loss of hair on the client's legs. The nurse suspects that the client may be experiencing A. venous stasis. B. varicose veins. C. thrombophlebitis. D. arterial insufficiency.

D. arterial insufficiency.

The nurse is planning to percuss the chest of an adult male client for diaphragmatic excursion. The nurse should begin the assessment by A. asking the client to take a deep breath and hold it. B. percussing upward from the base of the lungs. C. percussing downward until the tone changes to resonance. D. asking the client to exhale forcefully and hold his breath.

D. asking the client to exhale forcefully and hold his breath.

The nurse is planning to inspect the anal area of an adult male client. TO assess for any bulges or lesions the nurse shoi~lda sk the client to A. hold his breath. B. breathe deeply through his mouth C. breathe normally. D. bear down.,

D. bear down.

When the nurse places one hand flat on the body surface and uses the fist of the other hand to strike the back of the hand flat on the body surface the nurse is using A. firm percussion. B. direct percussion. C. indirect percussion. D. blunt percussion,

D. blunt percussion

The nurse suspects that a male client may have a hernia. The nurse should further assess the client for A. bruising at the site. B. urinary tract infection. C. cysts at the spermatic cord. D. bowel sounds at the bulge.

D. bowel sounds at the bulge.

The major artery that supplies blood to the arm is the A. radial artery. B. ulnar artery. C. posterior artery. D. brachial artery.

D. brachial artery.

The nurse is working with a community group to set up teaching programs to increase awareness among African-American women about preventive screening techniques for breast cancer. In the teaching program the nurse should plan to include A. local female physicians who work with cancer clients. B. hospital clinic workers from various racial backgrounds. C. nurses who work in outpatient centers. D. breast cancer patients of the same race.,

D. breast cancer patients of the same race.

While assessing the neurologic system of a confused older adult the nurse observes that the client is unable to recall past events. The nurse suspects that the client may be exhibiting signs of A. depression. B. anxiety. C. attention deficit disorder. D. cerebral cortex disorder.,

D. cerebral cortex disorder.

To arrive at a nursing diagnosis or a collaborative problem the nurse goes through the steps of analysis of data. After proposing possible nursing diagnoses, the nurse should next A. cluster the data collected. B. draw inferences and identify problems. C. document conclusions. D. check for the presence of defining characteristics.,

D. check for the presence of defining characteristics.

While assessing an adult client the nurse observes decreased chest expansion at the bases of the client's lu~lgsT. he nurse should refer the client to a physician for possible A. atelectasis. B. pneumonia. C. chest trauma. D. chronic obstructive pullnonary disease.,

D. chronic obstructive pullnonary disease.

The chambers of the eye contain aqueous humor which helps to maintain intraocular pressure and A. transmit light rays. B. maintain the retinal vessels. C. change refractory of the lens. D. cleanse the cornea and the lens.,

D. cleanse the cornea and the lens.

A common sign or symptom of depression in the elderly is A. rambling or incoherent speech. B. illusion or hallucinations. C. insomnia. D. cognitive impairment or pseudodementia.

D. cognitive impairment or pseudodementia.

When a client first entres the hospital for an elective surgical procedure, the nurse should perform an assessment termed A. entry. B. exploratory. C. focused. D. comprehensive.,

D. comprehensive.

During assessment of the genitalia of an adult male the client has an erection. The nurse should A. explain to the client that this often happens during an examination. B. cover the client's genitals and discontinue the examination. C. allow the client time to rest before proceeding with the examination. D. continue the examination in an unhurried manner.,

D. continue the examination in an unhurried manner.

The nurse is caring- for an adult female client when the client tells the nurse that she has had a clear discharge from her nipples for the past month. The nurse should ask the client if she has been taking A. antidepressants. B. antibiotics. C. insulin. D. contraceptives.

D. contraceptives.

An adult client tells the nurse that his eyes are painful because he left his contact lenses in too lon-g the day before yesterday. The nurse should instruct the client that prolonged wearing of contact lenses can lead to A. retinal damage. B. cataracts. C. myopia. D. corneal damage.

D. corneal damage.

While assessing the eyes of an adult client the nurse uses a wisp of cotton to stimulate the client's A. eyelid reflexes. B. refractory mechanism. C. lacrimal reflexes. D. corneal reflexes.,

D. corneal reflexes.

The nurse is assessing tlie neurologic system of n client who has spastic muscle tone. The nurse should explain to the client that spastic muscle tone is associated with impairment to the A. extrapyramidal tract. B. spinothalamic tract. C. posterior columns. D. corticospinal tract.

D. corticospinal tract.

The nurse is assessing the genitalia of a female client and detects a bulging anterior ~vallin the vagina. The nurse should plan to refer the client to a physician for A. stress incontinence. B. rectocele. C. tumor of the vagina. D. cystocele.

D. cystocele.

The pancreas of an adult client is located A. below the diaphragm and extending below the right costal margin. B. posterior to the left midaxillary line and posterior to the stomach. C. high and deep under the diaphragm and can be palpated. D. deep in the upper abdomen and is not normally palpable.

D. deep in the upper abdomen and is not normally palpable.

Hair follicles sebaceous glands, and sweat glands originate from the A. epidermis. B. eccrine glands. C. keratinized tissue. D. dermis.,

D. dermis.

While assessing the Achilles reflex in an 84-year-old client the nurse observes that the Achilles reflex is difficult to elicit. The nurse should A. refer the client to a physician for further evaluation. B. ask the client about injuries to the feet. C. determine whether the client is having any pain in the feet. D. document the finding in the client's record.,

D. document the finding in the client's record.

The nurse has tested an adult client's visual fields and determined that the temporal field is 90 degrees in both eyes. The nurse should A. refer the client for further evaluation. B. examine the client for other signs of glaucoma. C. ask the client if there is a genetic history of blindness. D. document the findings in the client's records.

D. document the findings in the client's records.

An adult female client visits the clinic for the first time. The client has many bruises around her neck and face and she tells the nurse the bruises are the "result of an accident." The nurse suspects that the client may be experiencing A. leukemia. B. diabetes mellitus. C. melanoma. D. domestic abuse.,

D. domestic abuse

Before examining the mouth of an adult client the nurse sliould first A. ask the client to leave dentures in place. B. don sterile gloves for the procedure. C. offer the client mouthwash. D. don clean gloves for the procedure.,

D. don clean gloves for the procedure.

If the nurse makes an error while documenting findings on a client's record the nurse should A. erase the error and make the correction. B. obliterate the error and make the correction. C. draw a line through the error and have it witnessed. D. draw a line through the error, writing "error" and initialing.,

D. draw a line through the error, writing "error" and initialing.

A client visits the clinic because she has missed one period and suspects she is pregnant. While assessing the client the nurse detects a sol~d, mobile, tender, unilateral adnexal mass. The client's cervix is soft. The nurse suspects that the client may be experiencing A. normal pregnancy. B. endometriosis. C. pelvic inflammatory disease D. ectopic pregnancy.,

D. ectopic pregnancy

The nurse is preparing to interview an adult client for the first time. The nurse observes that the client appears very anxious. The nurse should A. allow the client time to calm down. B. avoid discussing sensitive issues. C. set time limits with the client. D. explain the role and purpose of the nurse.

D. explain the role and purpose of the nurse.

The nurse is planning to interview a client who is being treated for depression. When the nurse enters the examination room the client is sitting on the table with shoulders slumped. The nurse should plan to approach this client by A. providing the client with simple explanations. B. offering to hold the client's hand. C. using a highly structured interview process. D. expressing interest in a neutral manner.

D. expressing interest in a neutral manner.

The size and shape of the breasts in females is related to the amount of A. glandular tissue. B. fibrous tissue. C. lactiferous ducts. D. fatty tissue.

D. fatty tissue.

Joints may be classified as cartilaginous synovial, or A. articulate. B. flexible. C. immobile. D. fibrous.

D. fibrous.

The nurse i~~preparintog examine the ears of an adult client with an otoscope. The nurse should plan to A. ask the client ro tilt the head slightly forward. B. release the auricle during the examination. C. use a speculum that measures 10 mm in diameter. D. firmly pull the auricle out up, and back.

D. firmly pull the auricle out, up, and back.

While reviewing a client's chart before seeing the client for the first time the nurse notes that the client has a diagnosis of pes planus. The nurse anticipates that the client has A. high arches. B. bunions. C. calluses. D. flat feet.

D. flat feet.

The nurse plans to assess an adult client for Homans' sign. The nurse should A. ask the client to remain standing for the procedure. B. place the hands on the client's thigh muscle. C. place the hands near the client's ankle. D. flex the client's knee then dorsiflex the foot.

D. flex the client's knee, then dorsiflex the foot.

A client visits the clinic and tells the nurse that after playing softball yesterday he thinks his knee is "locking up." The nurse should perform the McMurray7s test by asking the client to A. move from a standing to a squatting position. B. raise his leg while in a supine position. C. bend forward while trying to touch the toes. D. flex the knee and hip while in a supine position.,

D. flex the knee and hip while in a supine position.

A female client visits the clinic and complains to the nurse that her skin feels "dry." The nurse should instruct the client that skin elasticity is related to adequate A. calcium. B. vitamin D. C. carbohydrates. D. fluid intake.

D. fluid intake.

In some health care settings the institution uses an assessment form that assesses only one part of a client. These types of forms are termed A. progressive. B. specific. C. checklist. D. focused.,

D. focused.

If a male client is uncircumcised the glans of the penis is covered by the A. epididymis. B. frenulum. C. corona. D. foreskin.,

D. foreskin.

The result of a nursing assessment is the A. prescription of treatment. B. documentation of the need for a referral. C. client's physiologic status. D. formulation of nursing diagnoses.

D. formulation of nursing diagnoses.

A client visits the clinic and tells the nurse that his stools have been black for the past 3 days. The nurse should assess the client for A. gallbladder disease. B. colitis. C. polyps. D. gastrointestinal bleeding.

D. gastrointestinal bleeding.

While assessing the genitalia of a female client the nurse observes moist fleshy lesions on the client's labia. The nurse should refer the client to a physician for possible A. gonorrhea. B. herpes simplex virus infection. C. nabothian cysts. D. genital warts.

D. genital warts.

The nurse is caring for a client who is employed as a typist and has a family history of peripheral vascular disease. The nurse should instruct the client to reduce her risk factors by A. eating a high-protein diet. B. resting frequently. C. drinking large quantities of milk. D. getting regular exercise.

D. getting regular exercise.

While assessing the feet of an older adult client the nurse observes that the metatar~ophalan~ejaoli nt to the client's great toe is tender reddened, and painful. The nurse should refer the client to a physician for possible A. bunions. B. corns. C. hammer toe D. gouty arthritis.,

D. gouty arthritis.

The nurse is caring for a female client who has received a diagnosis of fibrocystic breast disease. The nurse has instructed the client about the disease. The nurse determines that the client needs further instructions when the client says she should avoid drinking A. regular coffee. B. regular tea. C. diet colas. D. grapefruit juice.

D. grapefruit juice.

During assessment of an adult client which of the following lifestyle practices would indicate to the nurse that the client may be at high risk for HIVIAIDS? A client who A. uses a condom on a regular basis. B. has multiple female partners. C. smokes marijuana occasionally. D. has anal intercourse with other males.,

D. has anal intercourse with other males.

An adult client visits the clinic and tells the nurse that she has had a sudden change in her vision. 'The nurse should explain to the client that sudden changes in vision are often associated with A. diabetes. B. the aging process. C. hypertension. D. head trauma.

D. head trauma.

Under normal circumstances the strongest stimulus in a human being to breathe is A. hypoxemia. B. hypocapnia. C. pH changes. D. hypercapnia.

D. hypercapnia.

During a client interviewthe nurse asks questions about the client's past health history. The primary purpose of asking about past health problems is to A. determine whether genetic conditions are present. B. summarize the family's health problems. C. evaluate how the client's current symptoms affect lifestyle. D. identifv risk factors to the client and his or her significant others,

D. identifv risk factors to the client and his or her significant others

While reviewing a client's chart before seeing the client for the first time - the nurse notes that the client has a diagnosis of Dupuytren's contracture. The nurse anticipates that the client will exhibit A. inability to turn the wrists. B. ulnar deviation of the hands. C. flexion of the distal interphalangeal joints. D. inability to extend the ring and little finger.

D. inability to extend the ring and little finger

An ongoing or partial assessment of a client A. focuses on a specific problem of the client. B. includes a comprehensive overview of all body systems. C. is usually performed by another health care worker. D. includes a brief reassessment of the client's normal body system.

D. includes a brief reassessment of the client's normal body system.

The nurse assesses a hospitalized adult client and observes that the client's jugular veins are fully extended. The nurse contacts the client's physician because the client's signs are indicative of A. pulmonary emphysema. B. diastolic murmurs. C. patent ductus arteriosus. D. increased central venous pressure.

D. increased central venous pressure.

During assessment of the vaginal area of an adult client the client tells the nurse that she has had pain in her vaginal area. The nurse should further assess the client for A. trauma. B. cancer. C. pregnancy. D. infection.,

D. infection.

While assessing an adult client the client tells the nurse that she "has had difficulty catching her breath since yesterday." The nurse should assess the client further for signs and symptoms of A. emphysema. B. cardiac disease. C. trauma to the chest. D. infection.,

D. infection.

The nurse is preparing to assess the abdomen of a hospitalized client 2 days after abdominal surgery. The nurse should first A. palpate the incision site. B. auscultate for bowel sounds. C. percuss for tympany. D. inspect the abdominal area.

D. inspect the abdominal area.

The nurse is assessing the abdomen of an adult client and observes a purple discoloration at the flanks. The nurse should refer the client to a physician for possible A. liver disease. B. abdominal distention. C. Cushing's syndrome. D. internal bleeding.

D. internal bleeding.

An African-American female client visits the clinic. She tells the nurse that she had her ears pierced several weeks ago and an elevated, irregular, reddened mass has now developed at the earlobe. The nurse should document a A. cyst. B. lichenification. C. bulla. D. keloid.,

D. keloid.

An adult client visits the clinic and tells the nurse that he has had excessive tearing in his left eye. The nurse should assess the client's eye for A. viral infection. B. double vision. C. allergic reactions. D. lacrimal obstruction.

D. lacrimal obstruction.

A client visits the clinic and tells the nurse that he has had lower back pain for the past several days. To perform Las2gue's test the nurse should ask the client to A. bend backward toward the nurse. B. lean forward and touch his toes. C. twist the shoulders in both directions. D. lie flat and raise his leg to the point of pain.,

D. lie flat and raise his leg to the point of pain.

While assessing the inguinal lymph nodes in an older adult client the nurse detects that the lymph nodes are approximately 3 centimeters in diameter, nontender, and fixed. The nurse should refer the client to a physician because these findings are generally associated with A. localized infection. B. systemic infection. C. arterial insufficiency. D. malignancy.,

D. malignancy.

A client visits the clinic for a routine physical examination. The nurse prepares to assess the client's skin. The nurse asks the client if there is a family history of skin cancer and should explain to the client that there is a genetic component with skin cancer especially A. basal cell carcinoma. B. actinic keratoses. C. squamous cell carcinoma. D. malignant melanoma.,

D. malignant melanoma.

The clavicles extend fron~th e acrornion of the scapula to the part of the sternum termed the A. body. B. xiphoid process. C. angle. D. manubriurn.

D. manubriurn.

While assessing an adult client's abdomen the nurse observes that the client's umbilicus is deviated to the left. The nurse should refer the client to a physician for possible A. gallbladder disease. B. cachexia. C. kidney trauma. D. masses.,

D. masses.

The nurse is planning to perform an eye and ear examination on an adulr client. After explaining the procedures to the client the nurse should A. ask the client to remain standing. B. show the client the otoscope. C. ask the client to remove his contact lenses. D. observe the client's response to the explanations.,

D. observe the client's response to the explanations.

While recording the subjective data of an adult client who complains of pain in his lower back the nurse should include the location of the pain and the A. cause of the pain. B. client's caregiver. C. client's occupation. D. pain relief measures.,

D. pain relief measures.

The nurse is planning to assess a client's abdomen for rebound tenderness. The nurse should A. perform this abdominal assessment first. B. ask the client to assume a side-lying position. C. palpate lightly while slowly releasing pressure. D. palpate deeply while quickly releasing pressure.

D. palpate deeply while quickly releasing pressure.

While assessing an adult client's head and neck the nurse observes asymmetry in front of the client's earlobes. The nurse refers the client to the physician because the nurse suspects the client is most likely experiencing alan A. enlarged thyroid. B. lymph node abscess. C. neurologic disorder. D. parotid gland enlargement.

D. parotid gland enlargement.

Transmission of sound waves in the inner ear is known as A. conducive hearing. B. tympanic hearing. C. neuromotor hearing. D. perceptive hearing.

D. perceptive hearing.

While performing a physical examination on a adult client the nurse can detect the density of an underlying structure by using A, inspection. B. palpation. C. Doppler magnification. D. percussion.,

D. percussion.

Before the nurse analyzes the data collected the nurse should A. determine collaborative problems with the health care team. B. group the data into clusters or groups of problems. C. generate possible hypotheses for the client's problems. D. perform the steps of the assessment process accurately.,

D. perform the steps of the assessment process accurately.

The external covering of the bone that contains osteoblasts and blood vessels is termed the A. cartilage. B. synovial membrane. C. connective tissue. D. periosteum.

D. periosteum.

The nurse assesses an adult male client's abdomen and observes diminished abdominal respiration. The nurse determines that the client should be further assessed for A. liver disease. B. umbilical hernia. C. intestinal obstruction. D. peritoneal irritation.

D. peritoneal irritation.

While assessing the anus of an adult client the nurse detects a peritoneal protrusion. The nurse should refer the client to a physician for possible A. anorectal fistula. B. polyps. C. prostate enlargement. D. peritoneal metastasis.,

D. peritoneal metastasis.

A client visits the clinic and complains of wrist pain. To perform Phalen's test the.nurse should ask the client to A. move the hand inward with the wrists straight. B. place both palms on the examination table. C. flex both wrists against resistance. D. place the backs of both hands against each other.,

D. place the backs of both hands against each other.

During assessment of an elderly male client the client tells the nurse that he has had difficulty urinating for the past few weeks. The nurse should refer the client to the physician for possible A. inguinal hernia. B. sexually transmitted disease. C, impotence. D. prostate enlargement.,

D. prostate enlargement.

A common error for beginning nurses who are formulating nursing diagnoses during data analysis is to A. formulate too many nursing diagnoses for the client and family. B. include too much data about the client in the history C. obtain an insufficient number of cues and cluster patterns. D. quickly make a diagnosis without hypothesizing several diagnoses.,

D. quickly make a diagnosis without hypothesizing several diagnoses.

The nurse has assessed the heart sounds of an adolescent client and detects the presence of an Sj heart sound at the beginning of the diastolic pause. The nurse sllo~~ilnds truct the client that she should A. be referred to a cardiologist for further evaluation. B. be examined again in 6 months. C. restrict exercise and strenuous activities. D. recognize that this finding is normal in adolescents.

D. recognize that this finding is normal in adolescents.

Straight movements of the eye are controlled by the A. lacrimal muscles. B. oblique muscles. C. corneal muscles. D. rectus muscles.

D. rectus muscles.

An adult client visits the clinic and tells the nurse that she has been experiencing frequent nosebleeds for the past month. The nurse should A. ask the client if she has had recent oral surgery. B. assess the client's nasal passages for blockage. C. ask the client if she is a smoker. D. refer the client for further evaluation.

D. refer the client for further evaluation.

While assessing the ears of an adult client the nurse observes bloody drainage in the client's ear. The nurse should A. docunient the finding in the client's chart. B. determine whether a foreign body is present in the ear. C. assess the client for further signs of otitis media. D. refer the client to a physician.,

D. refer the client to a physician.

While auscultating an adult client's heart rate and rhythm the nurse detects a regular irregular pattern. The nurse should A. assess the client for signs and symptoms of pulmonary disease. B. document this as a normal finding. C. schedule the client for an EGG. D. refer the client to a physician.

D. refer the client to a physician.

A client has tested 20140 on the distant visual acuity test using a Snellen chart. The nurse should A. document the results in the client's record. B. ask the client to read a handheld vision chart. C. ask the client to return in 2 weeks for another examination. D. refer the client to an optometrist.

D. refer the client to an optometrist.

The cerebrospinal fluid cushions the central nervous system (CNS) provides nourishment to the CNS, and A. transmits impulses. B. coats the brain. C. regulates heart rate. D. removes wastes.,

D. removes wastes.

During a thorough spiritual assessment the nurse understands that the questions asked are designed to A. encourage the client to explore other religions. B. cause the client to question long-held beliefs. C. determine if the client and nurse have similar beliefs. D. reveal beliefs that might affect client care.,

D. reveal beliefs that might affect client care.

A client visits the clinic and tells the nurse that she has painful cracking in the corners of her lips. The nurse should assess the client's diet for 3 deficiency of A. vitamin C. B. fluoride. C. vitamin A. D. riboflavin.

D. riboflavin.

The nurse plans to instruct an adult female client with regular menstrual cycles who is not taking oral contraceptives, about breast selfexamination. The nurse should plan to instruct the client to perform breast self-examination A. during menstruation. B. on the same day every month. C. midway between the cycles. D. right after menstruation.,

D. right after menstruation

To palpate for tenderness of an adult client's appendix the nurse should beg& the abdominal assessment at the client's A. left upper quadrant B. left lower quadrant. C. right upper quadrant. D. right lower quadrant.,

D. right lower quadrant.

A dark-skinned client visits the clinic because he "hasn't been feeling well." To assess the client's skin for jaundice the nurse should inspect the client's A. abdomen. B. arms. C. legs. D. sclera.,

D. sclera.

The nurse is assessing the spine of an adult client and detects lateral curvature of the thoracic spine with an increase in convexity on the left curved side. The nurse suspects that the client is experiencing A. lordosis. B. arthritis. C. kyphosis. D. scoliosis.

D. scoliosis.

The nurse is preparing to assess the lymph nodes of an adult client. The nurse should instruct the client to A. lie in a supine position. B. lie in a side-lying position. C. stand upright in front of the nurse. D. sit in an upright position.

D. sit in an upright position.

The roof of the oral cavity of the mouth is formed by the anterior hard palate and the A. teeth. B. gums. C. muscles. D. soft palate.

D. soft palate.

The Glasgow Coma Scale measures the level of consciousness in clients who are at high risk for rapid deterioration of the nervous systetn. A score of 13 indicates A. deep coma. B. severe impairment. C no verbal response. D. some impairment.,

D. some impairment.

The nurse is instructing a group of high school students about risk factors associated with various skin cancers. The nurse should instruct the group that A. melanoma skin cancers are the most common type of cancers. B. African Americans are the least susceptible to skin cancers. C. usually there are precursor lesions for basal cell carcinomas. D. squamous cell carcinomas are most conlmon on body sites with heavy sun exposure.

D. squamous cell carcinomas are most conlmon on body sites with heavy sun exposure.

The ossicles contained in the middle ear include the malleus the incus, and the A. pars tensa. B. pars flaccida. C. umbo. D. stapes.,

D. stapes.

An adolescent client tells the nurse that her mother says she ginds her teeth when she sleeps. The nurse should explain to the client that grinding the teeth may be a sign of A. precancerous lesions. B. poor oral hygiene. C. malabsorption. D. stress and anxiety.

D. stress and anxiety.

. The nurse is ~lanningto conduct the Weber test on an adult male client. To perform this test the nurse should plan to A. strike a tuning fork and place it at the base of the client's mastoid process. B. whisper a word with two distinct syllables to the client. C. ask the client to close his eyes while standing with feet together. D. strike a tunlng fork and place it on the center of the client's head or forehead.

D. strike a tunlng fork and place it on the center of the client's head or forehead.

Connecting the skin to underlying structures islare the A. papillae. B. sebaceous glands. C. dermis layer. D. subcutaneous tissue.

D. subcutaneous tissue.

While assessing the musculoskeletal system of an adult client the nurse detects tenderness, warmth, and a boggy consistency of the client's knee. The nurse should refer the client to a physician for possible A. torn meniscus. B. malignancy. C. fracture. D. synovitis.

D. synovitis.

A client visits the clinic and tells the nurse that he is depressed because of a recent job loss. He complains of dull aching, tight, and diffuse headaches that have lasted for several days. The nurse should document the client's A. cluster headaches. B. tumor-related headaches. C. migraine headaches. D. tension headaches.,

D. tension headaches.

While assessing the head and neck of an adult client the client tells the nurse she has been experiencing sharp shoot~ngfa cial pains that last from 10 to 20 seconds but are occurring more frequently. The nurse should refer the client for possible A. cancerous lesions. B. arterial occlusion. C. inner ear disease. D. trigeminal neuralgia.,

D. trigeminal neuralgia.

An adult client visits the clinic and tells the nurse that he has been "spitting up rust-colored sputum." The nurse should refer the client to the physician for possible A. pulmonary edema. B. bronchitis. C. asthma. D. tuberculosis.

D. tuberculosis.

An older client visits the clinic accompanied by his daughter The daughter tells the nurse that her father has been experiencing severe headaches that usually begin in the morning and become worse when he coughs. The client tells the nurse that he feels dizzy when he has the headaches. The nurse refers the client for further evaluation because these symptoms are characteristic of a A. migraine headache. B. cluster headache. C. tension headache. D. tumor-related headache.

D. tumor-related headache.

Any new onset of incontinence in the frail elder should be investigated for A. prostatitis. B. stroke. C. fecal impaction. D. urinary tract infection.

D. urinary tract infection.

While performing a physical examination on an older adult the nurse should plan to A. complete the examination as quickly as possible. B. ask the client to change positions frequently. C. provide only minimal teaching related to health care. D. use minimal position changes during the examination.,

D. use minimal position changes during the examination.

To assess an adult client's skin turgor the nurse should A. press down on the skin of the feet. B. use the dorsal surfaces of the hands on the client's arms. C. use the fingerpads to palpate the skin at the sternum. D. use two fingers to pinch the skin under the clavicle.,

D. use two fingers to pinch the skin under the clavicle.

During an interview with an adult client the nurse can keep the interview from going off course by A. using open-ended questions. 8. rephrasing the client's statements. C. inferring information. D. using closed-ended questions.,

D. using closed-ended questions.

One characteristic of a nurse who is a critical thinker is the ability to A. form an opinion quickly. B. offer advice to clients C. be right most of the time. D. validate information and judgments.

D. validate information and judgments.

Whiie transilluminating the scrota1 contents in a male adult client the nurse does not detect a red glow. The nurse should refer the client to a physician for possible A. spermatocele. B. orchitis. C. hydrocele. D. varicocele.

D. varicocele.

While inspecting the skin color of a male client's legs the nurse observes that the client's legs are slightly cyanotic while he is sitting on the edge of the examination table. The nurse should refer the client to a physician for possible A. arterial insufficiency. B. congestive heart failure. C. Raynaud's disease. D. venous insufficiency.,

D. venous insufficiency.

An example of an objective finding in an adult client is A. a client's symptom of pain. B. family history data. C. genetic disorders. D. vital signs.

D. vital signs.

A client tells the nurse that she has difficulty seeing while driving at night. The nurse should explain LO the client that night blindness is often associated with A. retinal deterioration. B. head trauma. C. migraine headaches. D. vitamin A deficiency.

D. vitamin A deficiency.

The nurse is caring for an adult client who is in a cast because of a fractured arm. To promote healing of the bone and tissue the nurse should instruct the client to eat a diet that is high in A. whole grains. B. vitamin B. C. vitamin E. D. vitamin C.,

D. vitamin C.

A client tells the nurse that his grandmother had a diagnosis of osteomalacia. The nurse should instruct the client that to decrease the risk factors for osteomalacia the clients should have adequate amounts of A. vitamin E. B. riboflavin. C. beta-carotene. D. vitamin D.,

D. vitamin D.

The rich blood supply of the nose serves to A. help propel moist air to the body. 8. pcopel debris to the throat. C. filter large particles from the air. D. warm the inspired air.

D. warm the inspired air.

Because body mass index (BMI) is calculated using only height and weight the nurse knows that inaccurate findings would most likely occur in a client A. with diabetes. B. who is six feet tall. C. with osteoarthritis. D. who is a bodybuilder.,

D. who is a bodybuilder.


Conjuntos de estudio relacionados

NURS 126 Practice Assessment A 2023

View Set

Chapter 37: Nursing Care of the Child With an Infectious or Communicable Disorder

View Set

Chapter 6 Review True or False? — AP Psychology

View Set

Final Exam - International Marketing 480

View Set

Biology Paper 2 Questions, 6 +11 Paper 2s, Biology Markscheme Active recall

View Set

M08mcq Module Completion Quiz (Module 08 Structuring Data & Blockchain (Roadmap))

View Set

Types of Credit, Credit & Credit Cards

View Set